[ /tv/ /rf/ /vg/ /a/ /b/ /u/ /bo/ /fur/ /to/ /dt/ /cp/ /oe/ /bg/ /ve/ /r/ /mad/ /d/ /mu/ /cr/ /di/ /sw/ /hr/ /wh/ /lor/ /s/ /hau/ /slow/ /gf/ /vn/ /w/ /ma/ /azu/ /wn/ ] [ Main | Settings | Bookmarks | Music Player ]

No.55775 Reply
File: wallpaper-470685.jpg
Jpg, 236.96 KB, 960×530 - Click the image to expand
edit Find source with google Find source with iqdb
wallpaper-470685.jpg
Предлагаю задавать здесь любые вопросы, касающиеся математики.
Полезные ссылки:
LaTeX pastebin: http://mathbin.net/
Если требуется что-то посчитать: http://www.wolframalpha.com/

Предыдущий: >>43626
>> No.55780 Reply
>>55771
> запили мне вывод дифференцирования из пустого множества не приплетая геометрических понятий, предела
Чем тебе не нравится определение предела по Гейне?
>> No.55782 Reply
>>55780
Речь не об определении предела без геометрических понятий, а о такой уж значимости пустого множества - без сопутствующих вещей, вроде пределов и прочего-прочего, построить матан из него? Налицо преувеличение.
>> No.55786 Reply
>>55771
> Во-первых, с какого боку ты приплёл физиков?
Потому что только они любят это слово "формализм".
> парадокс
Парадоксы наивной теории множеств не стоит ставит в ряд с тем, что не укладывается в нашем обычном видении мира. То, по сути и по форме, логические противоречия, требующие отказаться от интуитивных методов вывода и ограничить свободное оперирование множествами.
> Правда, бывает ещё и ложный, ошибочный.
У Бурбаки была ZF. Никто не знает противоречива ли она, но почти каждый математик ей пользуется и сейчас.
> дифференцирования
Чего и где?
> не приплетая геометрических понятий
Их не существует. Геометрия - раздел алгебры.
> предела
Кто сказал, что я не могу его определить? И, кстати, подтвердить корректность определений построениями из пустого множества.
> Коммутативная алгебра – очевидность среди очевидностей
Простой вопрос на знакомство по самой распространенной и небольшой по материалу книжке:
Что получится, если взять точную последовательность и умножить модули тензорно на какой-то произвольный модуль, а гомоморфизмы из последовательности на единицу для него?
> как и нужды в ней особой нет
Жаль, что вы не встречались ни гомологической алгеброй, ни с топологией, ни алгебраической геометрией, ни современной дифференциальной геометрией.
> Пример
Ну, даже не знаю: спектр кольца, например. Из него потом пучки получают. Вообще, коммутативная алгебра нужна даже линейной алгебре, которая вам, по-видимому, ближе.
> а о такой уж значимости пустого множества - без сопутствующих вещей
Значит аксиом предметных теорий множеств тоже не видели? Если не углубляться, то они гарантируют данными только два множества: пустое и {пустое, {пустое}, {пустое,{пустое}}, {пустое,{пустое},{пустое,{пустое}}},...}. Больше множеств и "предметов" нет. Всякие числа получаются только из этих двух.
>> No.55837 Reply
File: 7a9937d48fea3522f7d55fb42fdcbcf3.jpg
Jpg, 148.93 KB, 620×715 - Click the image to expand
edit Find source with google Find source with iqdb
7a9937d48fea3522f7d55fb42fdcbcf3.jpg
Народ, я вот учуть на сис. админа, (ивт). Я спрашивал у преподавателя будет ли у меня в курсе математики теория групп, топология и т.д. Преподаватель сказал, что нет, да и вообще такому тупице как я, хотя бы интегралы осилить, причём не интегрирование, а именно интегралы(так рф замашки в /u убрать).

Итак, в кафедре математики уж точно есть IT'шники, есть ли резон всё же изучать самостоятельно более углубленные разделы математики, я слоу конечно, но до меня допирает? Конечно для личного интереса да, но есть ли толк в IT сфере этого?
>> No.55838 Reply
>>55837
ИМХО. Знание математического аппарата позволит понимать "почему так а не иначе" решаются многие классические задачи в IT, следовательно, желание изобрести велосипед с преферансом и дамами будет посещать тебя значительно реже, и качество твоей работы возрастет.
>> No.55845 Reply
>>55837
> математики теория групп, топология
Рекомендую начать с дискретной математики, её у вас там всерьёз изучают?
>> No.55847 Reply
>>55837
> более углубленные разделы математики,
> теория групп, топологи
Насмешил.
> хотя бы интегралы осилить, причём не интегрирование, а именно интегралы
Взятие интегралов к математике не относится.
>> No.55854 Reply
>>55847
> Взятие интегралов к математике не относится.
Смелое утверждение.
>> No.55856 Reply
>>55837
> но есть ли толк в IT сфере этого?
Нет, никакого. Лучше почитай "Конкретную математику".

>>55847
> Взятие интегралов к математике не относится.
Детектируйте этого господина и скрывайте. Он явно перечитал вербятки и арнольда.

Теперь главное, я бегло просмотрел тред, конечно. Сложилось впечатление, что какой-то первокурсник мехмата, посетивший 3 лекции НМУ опять страшает доброанонов. Дескать, есть теория групп, теория категорий, диф гем и т.д. И это, дети мои, очень сложно. А то, что вы учите - это протухшие яйца, которые никому в современном мире не нужны. Есть альтернативный взгляд на вещи.

Математикам: вам нужно развивать как vision, так и executive power. Для второго Вы должны освоить все энтрилевел вычислительные области: матан, матан-3, тфкп, групповые методы, в топологии уметь считать гомотопческие группы (там больше ничего счетного вроде нет?), алгебры Ли. То есть все, что можно посчитать. Любая область, где сначала слегка теории, а потом дохуя задач обязательна к изучению. По мере возрастания вычислительных скиллов, вы будете видеть более глубокие структуры. Постепенно, вы переходите на более компактный и удобный язык. На нем сложнее считать, но легче понимать. Так вы откроете свой vision. Теперь классический пример, правда из физики. Уравнения Максвелла.

В начале 19 века Фарадей открыл электромагнитные волны. Потом Максвелл написал свои уравнения, которые были математически очень сложны. Любой, кто умел их написать, получал позицию в универах. Потом их слегка переписывали Лоренц и Пуанкаре. Потом устоялась тензорная запись. Потом их стали писать как формы на многообразие (одна пара - это d^2 (форма) = 0, вторая и есть связь с энергией). Сейчас их рассматривают как далекие следствия из теории p-форм на многообразиях. Спорить о том, нужно ли учить силу Лоренца, - бесполезно. Современное описание нужно для струн, Лоренц нужен, чтобы плазму описать.

Нематематикам: "математика мозги в порядок приводит". Вы не можете в современные теории, потому что там требуется уметь считать. С другой стороны, вы можете какие-то первые шаги сделать в топологии (но она сразу на интересных задачах становится алгебраической и под силу только титанам, привет Перельману). И я бы советовал обратить внимание на простую геометрию. Да,на ту, что была в школе. Только дополните ее школьной алгеброй и разбирайтесь смело в конических сечения, например. Так же можете в логики и теорию множеств. Это даже на философском факультете читают для девочек.

Сумбурно написал, потому что это тысячу раз уже высказывалось, правда, мужами более умными, чем я. Добра!
>> No.55872 Reply
>>55856
> Детектируйте этого господина и скрывайте.
Бестактно как.
> Он явно перечитал вербятки и арнольда.
Нет, ни того, ни другого. Но оба дали много полезной информации.
> Сложилось впечатление, что какой-то первокурсник мехмата, посетивший 3 лекции НМУ опять страшает доброанонов.
Не запятнан мехматом. В НМУ ни разу не ходил.
> Дескать, есть теория групп, теория категорий, диф гем и т.д. И это, дети мои, очень сложно. А то, что вы учите - это протухшие яйца, которые никому в современном мире не нужны. Есть альтернативный взгляд на вещи.
Наоборот говорю: это всё просто, а вот вычислять всякие интегралы сложно.
> уметь считать гомотопческие группы
Вы с ума не сошли случаем?
> там больше ничего счетного вроде нет?
Гомологии вычисляются легче.
> По мере возрастания вычислительных скиллов
В математике нужно доказывать и находить новое, а не вычислять. Человек вместо однотипных вычислительных задач должен доказывать разные утверждения, так он повышает способность к поиску разных фактов с какой-то целью.
> теории p-форм на многообразиях
Когда техника стала теорией?
>> No.56160 Reply
Аноны, почему 0.(9) = 1?
>> No.56169 Reply
>>56160
А это вообще правда? Может быть, все же можно захуячить иррациональное число между ними?
>> No.56170 Reply
>>56169
Это правда, если бы было можно взять число х между ними, то число х бы имело 10чную запись и тогда либо запись оказалась бы равна 0.(9) либо отличалось бы от неё хотя бы в одной цифре, а число значит было бы меньше 0.(9).
>> No.56171 Reply
>>56160
Прежде всего, 0.(9) не является десятичной записью числа по принятым соглашениям. Если использовать интуитивное понимание соответствия числа записи, то они, тем не менее, оказываются равными, поскольку сумма ряда 0+0.9+0.09+... равна 1.
>> No.56174 Reply
>>56171
Разве? помойму как раз по принятым соглашениям 0.(9) = 1, ну или хотя бы потому что 1 = 3(1/3) = 3 0.(3) = 0.(9)
>> No.56177 Reply
>>56174
При построении десятичной записи, исходя из аксиом действительных чисел, либо определяя вещественные числа, предполагают, что запрещена запись вида a,99..., если говорить формально.
> 1 = 3(1/3) = 3 0.(3) = 0.(9)
Умножать бесконечные десятичные дроби столбиком нельзя.
>> No.56188 Reply
>>56160
А оно не равно. На википедии, кроме того, ошибка в доказательстве.
>> No.56205 Reply
Анон, пожалуйста, объясни такую вещь, как предел. Я в принципе понимаю, что это за зверь такой, но как их вычислять?
>> No.56214 Reply
>>56205
> Я в принципе понимаю, что это за зверь такой,
То есть объяснять само понятие не нужно?
> как их вычислять?
Нужно узнать основные нетривиальные пределы. Потом, используя основные свойства, совершать преобразования к ним. Также можно раскладывать выражение в ряд до некоторых степеней переменной.
>> No.56215 Reply
>>56214
> То есть объяснять само понятие не нужно?
Ну, я понимаю, что это стремление функции к какому-то определённом числу.
> Нужно узнать основные нетривиальные пределы. Потом, используя основные свойства, совершать преобразования к ним. Также можно раскладывать выражение в ряд до некоторых степеней переменной.
А есть ли более универсальный метод? И где можно найти годный учебник на эту тему?
>> No.56217 Reply
>>56215
> Ну, я понимаю, что это стремление функции к какому-то определённом числу
Как-то непонятно, что ты понимаешь. Ничего не сказал. Предел - вовсе не универсальное понятие. Существует куча различных определений и методов введения, вплоть до зазубривания точных формулировок для одномерного анализа.
Проще всего его определить через непрерывность, которая гораздо более важна, универсальна, хотя кое-где также пытаются зазубрить её частное определение для одномерного анализа, типа "Действительная функция ф, определенная на некотором подмножестве множества действительных чисел, называется непрерывной в точке а, являющейся предельной для области определения ф, если для любого положительного эпсилон существует положительное дельта, что для всех икс из области определения ф, модуль разности которых с числом а меньше дельта, модуль разности значения функции ф в точке икс и значения ф в точке а меньше эпсилон" - так читается полное определение, аналогичное есть для предела. Отличие только в том, что икс и а не должны совпадать. Впрочем, это определение может быть проиллюстрировано разными картинками с графиками функций, поэтому сильно осуждать не буду.
> А есть ли более универсальный метод? И где можно найти годный учебник на эту тему?
Пределы объективно нужно решать лишь, чтобы обеспечить успеваемость в вузе. Для приобретения навыка нужно использовать задачники, а не учебники.
Нового ничего не скажу: есть Демидович http://free-books.us.to/book/index.php?md5=968DD131639895457EA5ACDDA19F2C13 (хорошее место, где можно найти почти любую книжку).
В книжке Фихтенгольца http://free-books.us.to/book/index.php?md5=3A13172440BFFBC204911620778D0AC3 (теорию учить по нему ни в коем случае не следует) есть примеры.
>> No.56218 Reply
>>56217
> являющейся предельной для области определения ф
P.S. В случае непрерывности в ней должна быть определена функция, а в случае предела может быть не определена.
>> No.56220 Reply
>>56217
Няша, не подскажешь, как доказать, что
lim{x->3}x^2=9 по определению Коши?
>> No.56222 Reply
>>56220
Вот это самое плохое в таких определениях: абсолютно ненужная техника, отсутствие проникновения в топологические смыслы.
Тут нужно найти какую-нибудь зависимость дельты от эпсилона. Достаточно взять дельта 1/6, если эпсилон больше 1, и шестую часть от эпсилон, если эпсилон меньше 1.
В том, что такая зависимость подходит убедиться вовсе не сложно.
>> No.56302 Reply
>>56217
Спасибо, няша, добра тебе.
>> No.56320 Reply
>>56222
Как её находить, эту зависимость? Есть какой-нибудь алгоритм или хотя бы набор рекомендаций?
>> No.56323 Reply
>>56320
> Как её находить, эту зависимость
В общем случае — никак.
> Есть какой-нибудь алгоритм
Если бы был, давно научили.
> хотя бы набор рекомендаций?
Это дело очень сложное: требуется сообразительность и опыт занятий этим бессмысленным делом.

Нет, конечно, не всё так плохо, потому что сложные скорее, неразрешимые вещи вам не дадут с такой формулировкой, значит где-то есть решение. Я бы никогда не пытался доказать через предел по Коши, ибо сама эта вещь уже бессмысленна.
Этот пример можно доказать вполне очевидно так: тождественная функция (заданная выражение эф от икс равно икс) непрерывна (по определению). Произведение непрерывных функция — непрерывная функция, то есть возведение в квадрат непрерывно (как и все функции, задаваемые многочленами). Это, по определению, означает, что предел равен значению этой функции в той точке.
>> No.56347 Reply
>>56220

lim{x->x0}f(x)=A <=> ∀ε>0 ∃δ=δ(ε)>0 ∀x : 0<|x-x0|<δ => |f(x)-A|<ε
1. |x^2-9|=|x-3||x+3|
2. x->3 => |x+3| < 4+3=7
3. ∀δ 0<|x-3|<δ
4. ∀δ |x-3||x+3|<δ*7
5. Пусть ∀ε δ=ε/7
6. ∀ε |x-3||x+3|<ε
7. ∀ε |x^2-9|<ε

Так сойдёт?
>> No.56349 Reply
>>56347
Неправильно: контрпример - δ=8, x=10, 91>56.
> x->3 => |x+3| < 4+3=7
Вообще глупость.
>> No.56350 Reply
>>56349
> x=10
Так ведь x стремится к трём.
>> No.56351 Reply
>>56350
> Так ведь x стремится к трём.
Замечательная логика! Там есть «радиус стремимости»? То есть 10 — это слишком далеко, а вот если взять 1 или 0,5, то в самый раз — стремится. А вдруг не стремится? Когда начинает стремиться, вообще? А если задать другую зависимость, то радиус изменится, и что же делать?
> ∀ε>0 ∃δ=δ(ε)>0 ∀x : 0<|x-x0|<δ => |f(x)-A|<ε
Вроде бы почти правильное определение предела, где в нём говорится про «стремимость»?
Раз для каждого ε>0, то я могу взять ε=56. Ведь так?
Ты утверждаешь, что нашёл хороший метод вычисления δ. Так, по твоим определениям, δ=ε/7=8 подходит.
Раз всё это правильно, то всякое число x из объединения интервалов (-5; 11)-{3} удовлетворяет |x^2-9|<56, но x=10 не подходит, да и вообще всякое число из (9; 11) не удовлетворяет наше неравенство.
Извиняюсь за кучу сарказма.
>> No.56352 Reply
>>56351
Значит, я не понимаю смысла пределов. Объясни, пожалуйста.
>> No.56355 Reply
File: 00103.jpg
Jpg, 25.86 KB, 440×247 - Click the image to expand
edit Find source with google Find source with iqdb
00103.jpg
>>55837
> Народ, я вот учуть на сис. админа, (ивт)
> Я спрашивал у преподавателя будет ли у меня в курсе математики теория групп, топология и т.д
Проиграл с подливой. У вас даже скорее всего теории графов или ТА не будет, какая топология.
> да и вообще такому тупице как я, хотя бы интегралы осилить, причём не интегрирование, а именно интегралы
Ну, если начать произносить такие магические слова, как "мера Лебега (или Хаусдорфа, не суть)", "измеримое множество", "интегрирование по сигма-аддитивной мере" и так далее, то это одно. А если "возьми интеграл бесплатно без смс получи 5 автоматом", то это совсем другое. Хотя интеграл разные бывают. Бывают двойные/тройный по хитровыебанной области с хитровыебанной подинтегральной функцией. Я даже в Демидовиче такие, помнится, видел.
> Итак, в кафедре математики уж точно есть IT'шники, есть ли резон всё же изучать самостоятельно более углубленные разделы математики
Абсолютно никакого. Во-первых, это занятие лично для тебя лишено смысла, нет практической отдачи. Во-вторых, ты тупо не осилишь хотя бы введение в алгебраическую топологию.
> но есть ли толк в IT сфере этого
Да что же ты делаешь, я опять на всю комнату проиграл.
>>55856
> Математикам: вам нужно развивать как vision, так и executive power
Shut up and calculate во все поля, писал явно физик.
> Современное описание нужно для струн,
Ээ, струны - это уже далёко забрались, батенька. Я в рашке сходу не назову людей или подразделение, которое (-ые) этим всерьёз занимаются, пишут статьи и тд. Шестимерные многообразия Калаби-Яу - это, конечно, очень увлекательно, но это надо сильно специфичный мозг иметь, чтобы с этим работать и кайфовать. Даже дифгем на простом уровне осознать - это не каждому дано.
> Нематематикам: "математика мозги в порядок приводит".
...или позволяет показать полное их отсутствие.
> Вы не можете в современные теории, потому что там требуется уметь считать.
Я бы назвал это загадочным "математическим мышлением". Но в целом да.
> Так же можете в логики и теорию множеств. Это даже на философском факультете читают для девочек.
Да ну? Что вот прямо и теорему Гёделя им доказывают? И про секвенциальное исчисление рассказывают?
> Сумбурно написал, потому что это тысячу раз уже высказывалось, правда, мужами более умными, чем я. Добра!
Сумбурно, но в целом по делу. Два чаю за счёт доброчана.
>>56205
Здесь должны набижать няши-функанщики и пояснить по хардкору про разные виды сходимости, сети и тд. Я набижал, но мне лень.
>>56351
Есть такая штука - окрестность. Так вот, если вне любой окрестности точки L лежит не более, чем конечное число точек последовательности Xn, то говорят, что L - предел Xn. Как-то так нам давали на 1 курсе.
> Раз для каждого ε>0, то я могу взять ε=56. Ведь так?
Можешь, только зачем городить огород там, где не надо?
>> No.56360 Reply
File: calc.pdf
Pdf, 0.09 KB, 595×842 - Click the image to get file
calc.pdf
>>56352
Приложил файл. Если будут вопросы — задавай.
>>56355
> Есть такая штука - окрестность.
И что? Есть даже две (по Бурбаки и традиционная).
> Так вот, если вне любой окрестности точки L лежит не более, чем конечное число точек последовательности Xn, то говорят, что L - предел Xn
Ну, это правда. И что?
> где не надо?
А где надо? Вообще, значит есть, где надо, и есть, где не надо? Прочитай про базы топологий, в файле есть.
>> No.56361 Reply
>>56360
P.S. Забыл, что там такое: есть определение. Опр. Отображение топологических пространств называется непрерывным тогда и только тогда, когда прообраз всякого открытого множества открыт.
Упр. Докажи, что функция непрерывна тогда и только тогда, когда она непрерывна в каждой точке области определения.
>> No.56362 Reply
File: 1.JPG
Jpg, 42.90 KB, 806×454 - Click the image to expand
edit Find source with google Find source with iqdb
1.JPG
>>56360
> И что? Есть даже две (по Бурбаки и традиционная).
Кот знает, что там определяли товарищи Бурбаки, я знаю единственное определение окрестности в функане и его проекцию на матан (и R, соотв.), мне его вполне достаточно.
> Ну, это правда. И что?
Ну так и пляши от определения, я тогда не понимаю, какие вопросы.
> А где надо? Вообще, значит есть, где надо, и есть, где не надо?
Это приходит с опытом, которую небыдло любит называть "математической грамотностью". В случае обычного предела в R - никаких ε=56 брать не надо. Вообще, на мой взгляд, чтобы доказать по определению в тривиальном примере >>56220, достаточно начертить график и сказать "из геометрических соображений очевидно".
> Прочитай про базы топологий, в файле есть.
Это к чему? не лекции ли это по топологии упоминавшегося вербицкого, случаем?
Я про это всё знаю, более того, когда будет нужно, я расскажу, почему в нормированных пространствах нельзя обойтись без сетей, например. Ну это если найдётся благодарный и понимающий слушатель :3
>> No.56363 Reply
File: 3.png
Png, 126.42 KB, 900×985 - Click the image to expand
edit Find source with google Find source with iqdb
3.png
>>56361
> Опр. Отображение топологических пространств называется непрерывным тогда и только тогда, когда прообраз всякого открытого множества открыт.
На самом деле, можно дать, ээх, дай Бог вспомнить, четыре эквивалентных определения непрерывности в топологических пространствах.
> Докажи, что функция непрерывна тогда и только тогда, когда она непрерывна в каждой точке области определения.
Эм, не понял. Дело в том, что функция всегда непрерывна где-то. По дефолту, она непрерывна в своей области определения (опустим пока ТФКП). Тогда мы опускаем остаток и говорим просто, что "функция непрерывна". Так что это не упражнение, а скорее удобное словесное обозначение.
>> No.56364 Reply
>>56362
> там определяли товарищи
Всякое множество, содержащее открытое, содержащее точку.
> Ну так и пляши от определения,
Не буду. Это определение плохое, оно мне не нравится.
> небыдло любит называть "математической грамотностью"
Небыдло любит это называть базой: все открытые шары с радиусом меньше 56 — база.
> "из геометрических соображений очевидно".
Нет, не достаточно.
> не лекции ли это по топологии упоминавшегося вербицкого
Нет, это мат. анализ Львовского (1 семестр).
> Я про это всё знаю, более того, когда будет нужно, я расскажу, почему в нормированных пространствах нельзя обойтись без сетей
Рассказывай. Я буду слушать.
>>56363
> можно дать,
Такое дают.
> четыре эквивалентных определения непрерывности в топологических пространствах
Определение непрерывного отображения топологических я знаю только это.
> Дело в том, что функция всегда непрерывна где-то.
Проверяют непрерывность при фиксированных топологиях.
> По дефолту, она непрерывна в своей области определения
Всякая функция из антидискретного топологического пространство в дискретное, принимающая больше одного значения, не является непрерывной. Всякое множество может быть снабжено этим топологиями. Не понимаю, что ты говорил.
> Так что это не упражнение, а скорее удобное словесное обозначение
у нас есть два определение
1) определение непрерывного отображения топологических пространств,
2) определение непрерывности отображения топологических пространств какой-то точке.
Упр. 1 тогда и только тогда, когда 2 в каждой точке. Да, оно устное, но зато полезное. Тем более, тот, к кому я обращался, ещё плохо знает определение предела через эпсилон-дельта формализм.
>> No.56379 Reply
>>56360
Спасибо за файл, сарказм-кун. Ты няша.
>> No.56382 Reply
File: 129157543497.jpg
Jpg, 24.26 KB, 447×540 - Click the image to expand
edit Find source with google Find source with iqdb
129157543497.jpg
>>56352
Смысл предела в том, что это, как ни странно, предел. Это если именно хочешь понять его - функция стремится к определённому значению y0, допустим, при аргументе, стремящемуся к x0. Легко представить на гиперболе, взяв односторонний предел: начерти график, и посмотри, к чему стремится функция при х->0+. Ты наверное уже заметил - к бесконечности. Так вот предел в данном случае - бесконечность. Выглядит так: lim{x->0+}1/x=∞
0+ - знак плюса потому что мы взяли предел справа - то есть аргумент стремится справа к 0, а функция к бесконечности. Взять слева, и: lim{x->0-}1/x=-∞. Начерти гиперболу и сам поймешь почему.
Главное чтоб не путалось значение предела и значение самой функции при аргументе, стремящемуся к какому-либо х0. Функция стремится к, а предел равен бесконечности - в данном случае.


Да, если односторонние пределы при аргументе, стремящемуся к х0 не совпадают, то у функции предел в этой точке не существует. Ибо слева это минус бесконечность, справа - плюс бесконечность, этакий разрыв. А следовательно мы не можем сказать, к чему стремится функция при таком х.
>> No.56409 Reply
>>55837
(раз уж начали отвечать аж отсюда, то присоединюсь)
Смотря кем ты хочешь быть. Смотри по ходу дела чему именно вас будут учить - погляди название предметов у старших курсов хотя бы. Если ты будешь заниматься чем-то близким к "фундаментальной науке" и так далее, то можешь слушаться >>55856-куна.
Если же тебе больше "на практику" и "для сисадминства", то слушай >>55845-куна. Как только узнаешь что такое P, NP (и EXP), то можешь начинать. А делать это лучше с графов, затем посмотреть какую-нибудь теорию кодирования, может - распознавание образов. Оптимизационные задачи (всё то, что можно назвать "теоретической кибернетикой"). Там глядишь дойдёшь ещё докуда, где случится высший матан. Его-то и начнёшь копать. Не брезгуй википедией (но и не доверяй ей на 100%): её система перекрёстных ссылок очень помогает тем, кого она не запутывает.
А, да... в любом случае сиди и читай computer science (и я про раздел науки, а не про журнал... хотя журнал тоже, наверное, можешь).

>>56349
Справедливо. С формальной точки зрения. Но всё же зачем пугать и путать человека не очень разбирающегося в пределах? Для него хватит и "асимптотического" (выпендирлся, сказал умное слово) случая.

>>56355
> "интегрирование по сигма-аддитивной мере"
Мне честно интересно, хоть кто-нибудь кроме изучал интеграл Лебега через сигма-аддитивную меру? Или это было сказано так - "для понта"?
> Я даже в Демидовиче такие, помнится, видел.
Демидович-номер-3940.джву?

>>56364
> Определение непрерывного отображения топологических я знаю только это.
ruwiki://Непрерывное_отображение#.D0.AD.D0.BA.D0.B2.D0.B8.D0.B2.D0.B0.[...]D1.8F
Аватаркофаг-кун говорил о первых четырёх определениях отсюда. Первое - которое дал ты. Второе - симметричное ему, но с заменой "открытое" на "замкнутое". Третье - звучит странно, но если рассматривать на R, то мы получим просто эпсилон-дельта определение непрерывной функции. Четвёртое определение же, просто поточечное. "Если функция непрерывна в каждой точке (давайте здесь сошлёмся на какого-нибудь ещё куна с его определением непрерывности в точке; да хотя бы на >>56382, пусть и нехорошо), то она непрерывна в том смысле, который даёт остальные три определения (и наоборот)" - вот его смысл в пересказе.
Эквивалентность этих четырёх определений как-то там доказывается.
> опустим пока ТФКП
Вот так всегда. На самом деле, возвращаясь после ТФКП на обычную вещественную ось начинаешь чувствовать некий дискомфорт от появления точек плюс и минус бесконечность: когда уже узнал, что на самом деле точка "бесконечность" одна-единственная, принимать частный случай, в котором этих точек две... ну-у-у... как-то неправильно выглядит, что ли?
>> No.56433 Reply
File: Akashi.jpg
Jpg, 110.87 KB, 443×650 - Click the image to expand
edit Find source with google Find source with iqdb
Akashi.jpg
>>56364
Много умных слов, мало смысла. Есть непрерывность в области - это непрерывность во всех точках области, кроме, быть может, границы. А есть непрерывность в конкретной точке. Всё, пожалуй тут нечего больше рассусоливать.
>>56409
> А делать это лучше с графов, затем посмотреть какую-нибудь теорию кодирования, может - распознавание образов
Добавлю лишь, что всё это читают на матфаке в государственном вузе твоего города. Приходи на спецкурсы и слушай.
> А, да... в любом случае сиди и читай computer science (и я про раздел науки, а не про журнал... хотя журнал тоже, наверное, можешь).
Не люблю я это понятие, сильно оно расплывчатое. Одно дело кодить, заниматься программной архитектурой и тд, а совсем другое - это какая-нибудь теория типов, лямбда исчисление, параллельные процессы, криптология и тд. Я обращаюсь ко всем, слово computing здесь может вводить в заблуждение. Это безусловно связано с компьютерами и программированием, но это очень сложная математика на самом деле. В той же криптологии есть булевозначный анализ и теория Галуа, без них совсем никак и никуда.
> Мне честно интересно, хоть кто-нибудь кроме изучал интеграл Лебега через сигма-аддитивную меру? Или это было сказано так - "для понта"?
Я, например, изучал. Могу назвать профессора и вуз. Сказано было скорее действительно для понта.
> Демидович-номер-3940.джву?
Верю на слово. Факт в том, что я точно помню, что такие были.
> Аватаркофаг-кун говорил о первых четырёх определениях отсюда
Да, все нижеследующие пояснения верны, так и есть. Называется это в духе "лемма об эквивалентных определениях непрерывности". Доказывается почти очевидно, даже в терминах фильтров и сетей.
> Вот так всегда. На самом деле, возвращаясь после ТФКП на обычную вещественную ось начинаешь чувствовать некий дискомфорт от появления точек плюс и минус бесконечность: когда уже узнал, что на самом деле точка "бесконечность" одна-единственная, принимать частный случай, в котором этих точек две... ну-у-у... как-то неправильно выглядит, что ли?
Ну, ТФКП всё таки сильно шаблон рвёт. Неограниченностью тригонометрических функций или переодичностью экспоненты. Или теоремой Коши. Или нуохуетьблядь способами подсчётов интегралов через вычеты. А ещё счётнолистными функциями, типа логарифма. После такого, в R как-то совсем всё тривиально.
А насчёт бесконечности как раз нормально. Как-то быстро переключается туда-обратно.
>> No.56440 Reply
>>56355
> Я в рашке сходу не назову людей или подразделение, которое (-ые) этим всерьёз занимаются, пишут статьи и тд
ИТЭФ, ОТФ ФИАН и ИЯИ (там, правда, браны)
> Да ну? Что вот прямо и теорему Гёделя им доказывают? И про секвенциальное исчисление рассказывают?
Мне как-то одна девочка с филфака МГУ рассказывала, что на первом курсе у них было доказательство Кантора несчетности иррациональных чисел, но главное, там самая пиздатая кафедра - кафедра мат. логики.
Не знаю, как тебе, но мне, с моим физическим бэкграундом, они точно нос утрут, думается мне. Хотя они и не знают, чем равен интеграл exp(-x^2) по всей оси.
Что такое секвенциальное исчисление я не знаю.
>> No.56444 Reply
>>56409
> (давайте здесь сошлёмся на какого-нибудь ещё куна с его определением непрерывности в точке; да хотя бы на >>56382, пусть и нехорошо)
Я не понял.жпг
Что нехорошо? Если определение непрерывности, то это определение всего-навсего предела было. Непрерывность бы определялась равенством значения функции существующему пределу в какой-либо точке.
> Верю на слово. Факт в том, что я точно помню, что такие были.
А что особенного в двойных/тройных? Там все задачки в многомерном анализе такие. В курсе матана он проходится со всем остальным - включая векторный анализ и контурное интегрирование - где-то на первом курсе.
>> No.56462 Reply
>>56440
> ИТЭФ, ОТФ ФИАН и ИЯИ (там, правда, браны)
Здорово. Как вообще теория струн среди физического коммьюнити в рашке воспринимается?
> Мне как-то одна девочка с филфака МГУ рассказывала, что на первом курсе у них было доказательство Кантора несчетности иррациональных чисел
Для доказательства этой теоремы хватит школьной программы + пары теорем. Там же всё на пальцах, другое дело, что мне было бы крайне интересно на экзамене у этих вот девочек спросить, а каков смысл того, что вы в итоге из этой теоремы получили.
> Не знаю, как тебе, но мне, с моим физическим бэкграундом, они точно нос утрут, думается мне
Вряд ли, впрочем я не знаю, как у физиков дела с матлогикой обстоят.
> Хотя они и не знают, чем равен интеграл exp(-x^2) по всей оси.
Если тервер не проходят, то не знают. С другой стороны, это знание гораздо более частное и узкое, нежели вышеупомянутая т. Кантора.
>> No.56467 Reply
>>56462
> а каков смысл того, что вы в итоге из этой теоремы получили.
Не люблю такие вопросы. Чтобы ответить на них, студент должен догадаться, что от него хочет услышать препод, и сформулировать свой ответ с помощью таких слов, чтобы преподу понравилось.
>> No.56470 Reply
>>56462
Бро, для тебя это это интеграл из теор.вера, а я его часто вижу в квантах и стат. физике.
> Вряд ли, впрочем я не знаю, как у физиков дела с матлогикой обстоят.
Мне хватает обычной логики. Даже не знаю, что там может быть вумного в этом разделе.
> Здорово. Как вообще теория струн среди физического коммьюнити в рашке воспринимается?
Нормально воспринимается. Некоторые струнные модели применяются сейчас в сверхпроводимости, то есть у твердотельщиков (ads\cft corresponding). А у этих пацанов все по хардкору, как Ландау завещал. Другое дело, что теория струн разводит математику - огогого, а дает нихуя. Квантовая теория поля, например, тоже использует неибический мат. аппарат (насколько я слышал, интегралы по Боголюбову до сих пор не формализованы, а физики уже 30 лет по ним считают), но дает фантастическое согласование с экспериментом. Такие вот общие места, но ты и сам, скорее всего, это знаешь.

А по делу если, то гугли Цейтлина и Мецаева.
>> No.56476 Reply
>>56433
> в терминах фильтров и сетей
Зачем нужны тебе фильтры? И ты говоришь про ε-сети? Зачем они тоже нужны? Неужели в терминах фильтров и ε-сетей можно что-то описать нормальное?
>> No.56479 Reply
File: c9f7cf8a4c2fb1886...
Jpg, 693.62 KB, 2000×1500
edit Find source with google Find source with iqdb
c9f7cf8a4c2fb1886a23eebf2766970d.jpg
File: Konachan.com - 13...
Jpg, 1111.48 KB, 1680×1193
edit Find source with google Find source with iqdb
Konachan.com - 132609 aqua_eyes aqua_hair barefoot bed beek blush breasts harem hatsune_miku long_hair tattoo twintails vocaloid.jpg
File: f66dbc915a22da6c5...
Png, 1452.21 KB, 1600×900
edit Find source with google Find source with iqdb
f66dbc915a22da6c50b894c67f96b32f.png

Няши, нужна ваша помощь по матфизике.

Требуется решить задачу Римана аналитически.

Прикладываю вам учебник и программу, которая решает случаи для разрыва по давлению и скорости (ударная волна и тангенциальный разрыв).

Пытаемся с одним аспирантом заставить работать третий случай — контактный разрыв (по плотности). Не хочет расползаться решение и всё тут! Стоит на месте как вкопанное, как бы не меняли параметры t.

Если не сложно, объясните вкратце, как нам допилить третий тест, исходя из того, что книгу мы (в частности, я) не читали (слишком там много лишнего, а нам нужно всего-лишь для идеального газа рассмотреть случай).

http://db.tt/598gkh1y - Куликовский, Погорелов, страница 160.
http://dl.dropbox.com/u/21229436/2012_2/Coursework/riemannCsharp.zip - полный исходник с проектом.
http://dl.dropbox.com/u/21229436/2012_2/Coursework/r/r.zip - сама программа и исходник.
>> No.56495 Reply
>>56479
Я исходники вечером посмотрю, щас лениво разбираться. У меня вопрос такой, как вы посчитали аналитическое решение? Задача с разрывом - только численные методы же, не? Во-вторых, вы считаете сначала скорость ударной волны и волны разрежения, а потом пересчитываете давление, плотность и (какой у вас 3 параметр?) в облостаях за\перед волнами. Так?
>> No.56515 Reply
File: 2012-03-20-485499...
Jpeg, 513.66 KB, 1300×1300
edit Find source with google Find source with iqdb
2012-03-20-485499.jpeg
File: dd188d25cc82b3000...
Jpeg, 132.44 KB, 600×600
edit Find source with google Find source with iqdb
dd188d25cc82b3000e0f0a26b9d5b257.jpeg
File: N5mRV.jpg
Jpg, 65.00 KB, 720×540
edit Find source with google Find source with iqdb
N5mRV.jpg

>>56495
Я сам пока очень плохо в этом разбираюсь, но здесь имеется в виду, что аналитическое решение имеется для базовых задач, с фиксированными начальными условиями. Универсальное решение получить аналитически невозможно.

Вот мои численные решения для каждого случая:
http://i.imgur.com/PynW6.gif (разрыв давления)
http://i.imgur.com/sFHMI.gif (тангенциальный разрыв или разрыв по скорости)
http://i.imgur.com/3D0EI.gif (контактный разрыв или волна разрежения)
Последний пункт надо получить аналитически.

Всего параметров сплошной среды у нас 3 (базовых): давление (p), плотность (\rho), скорость или импульс (v, impulse = \rho * v).
Так же имеется газовая адиабата (gamma), равная 5/3.
Область расчёта это [0;1], ровно посередине происходит деление параметров сплошной среды. Таким образом, левая и правая стороны означают соответственно параметры слева и справа от границы.

Решение ищется в виде автомодельной переменной xi: ruwiki://Автомодельное_решение
Мы её подбирали сами так, чтобы граница была ровно посередине (x = 0.5), при t -> 0. Почему-то получилось, что xi = (-1.5 + i*dx ) / t, откуда там взялось "-1.5", я не догоняю пока что.
Переменные ADDTIME - шаг по времени; a = -1.5 -- какой-то неизвестный коэффициент в xi, t -- начальный момент времени решения.
>> No.56540 Reply
>>55775
Доброго дня кафедре.
Пишет вам бывший я-философ-математика-не-нужна-кун.

Решил восстанавливаться в просраный универ (учился и буду доучиваться на кодера) - понял, что чтобы доучиться на годного программера люто, бешено необходимы матан, матстат, теорвер и дискретная математика. И все бы хорошо, но все два курса, когда это читали, я прогулял по конференциям по гуманитарным наукам, за что не раз был вытягиваем лично деканом факультета философии из лап математиков.

Реквестирую годных учебников по матану, теорверу и матстату.
>> No.56541 Reply
>>56540
Матан - Письменный, "конспект лекций", потом, примерно через полгода - "Сборник лекций по математическому анализу", Садовничий, серия "Современный учебник"; задачник - Демидович. Линал - Поздняк, задачник - Проскуряков. Дискретная математика - Просветов, а потом ещё кто-нибудь.
>> No.56559 Reply
File: 1301869321_ilin_v.a._lineinaya_algebra.jpg
Jpg, 14.50 KB, 200×297 - Click the image to expand
edit Find source with google Find source with iqdb
1301869321_ilin_v.a._lineinaya_algebra.jpg
>>56541
Если ты об одном из авторов этого классического нелюбимого мной за отвратительный школярский стиль учебника, то позволь фикс: Позняк. Учебниг довольно тонкий, поэтому для быстрой подготовки вполне подойдёт.
Капча хороша, но символ я не проспал.
>> No.56562 Reply
File: 1334521795821.png
Png, 0.82 KB, 300×20 - Click the image to expand
edit Find source with google Find source with iqdb
1334521795821.png
>>56559
> Учебниг довольно тонкий
Фикс: Учебник. Ошибся при переделывании слова книга.
А вот ещё отклеившаяся с того поста капча.
56559-кун
>> No.56563 Reply
Я мудак, кусок кретина, никчемное чмо. Прослушал весь второй семестр алгебры, ибо на первых двух занятиях и лекциях ни черта не понял. Но тут скоро контрольные две и коллоквиум и я понял, что охуенно надо все наверстывать. Темы от понятия евклидовых пространств(нер-ва коши-буняковского и т.д.) до Теоремы Кеми-Гамильтона.

Так вот, для этих целей подойдет Кострикин с его трехтомником?
>> No.56564 Reply
Посоны, что вы думаете про книгу Р. Куранта "Что такое математика?"?
По-моему отличная книга для тех, кто желает ознакомиться с самой сутью математики, научиться действительно её понимать и мыслить математически. А книги вроде Письменного просто учат решать задачки. По крайней мере, такое у меня впечатления сложилось после первых 50ти страниц, дальше я не осилил.
>> No.56565 Reply
>>56564

Понятие математика воспринимаю в оригинальном ключе, поэтому мои мысли могут показаться странными.

Посмотрел содержание и подряд несколько страниц в некоторых случайно выбираемых местах. Едва ли в книге излагается что-либо, что я бы стал относить к математике. Если написанное в ней читатель станет связывать с математикой, то это может привести к тому, что он запутается.
>> No.56566 Reply
>>56515
Бро, а тебе это в вузике задали? Что за вузик такой? Может оказаться, что мы в одном месте учимся. А если нет, то еще лучше. Потому что мне тоже надо эту задачку запилить.
>> No.56568 Reply
>>56563
> Так вот, для этих целей подойдет Кострикин с его трехтомником?
Подходит, почему же нет? Кострикина я ещё в школе читал. Говорят, конечно, что её не любят, но плохого в том, что читал не нашел. Могу сказать, что она слишком техническая.
Мне кажется, из русских книжек Винберг для первоизучения - в самый раз. Если имеется бурбакистская жилка, то можно Ленга почитать.
> Кеми
Две ошибки.
>>56564
> Р. Куранта "Что такое математика?"?
Хорошая книга.
> тех, кто желает ознакомиться с самой сутью математики,
С сутью творческого математического метода, использованного на школьной математике. Было бы полной глупостью искать там фундаментальные математические понятия (но не концепции математического естествознания).
> А книги вроде Письменного просто учат решать задачки
Письменный ненавидим мною.
>> No.56571 Reply
>>56566
Волгу же. Ололо дианон легион
>> No.56576 Reply
File: Screenshot-Yojouhan Shinwa Taikei - 01 RAW (CX 848x480 H.264 AAC).mp4-1.png
Png, 176.24 KB, 848×480 - Click the image to expand
edit Find source with google Find source with iqdb
Screenshot-Yojouhan Shinwa Taikei - 01 RAW (CX 848x480 H.264 AAC).mp4-1.png
>>56563
Неплохие лекции по линейной алгебре.
http://matematika.phys.msu.ru/stud_gen/6 - бадьина(овчинникова тоже ничего).
>> No.56590 Reply
>>56571
Как славно! У меня другой вузик и та же самая задача. Верней, надо запрогать точное решение + метод Годунова + метод второго порядка. Правда, я еще не садился.(Верней, написал неработающее аналитическое решение, которое сам не понял). Можешь свои контакты оставить?
>> No.56608 Reply
>>56568
А какие книги посоветуешь, чтобы ознакомиться с фундаментальными математическими понятиями?
>> No.56610 Reply
>>56608
В книжки на английском языке можешь?
>> No.56612 Reply
>>56610
Ну могу попробовать, если книга годная
>> No.56615 Reply
File: Lawvere_F._W.,_Rosebrugh_R._-_Sets_for_Mathematics(2003)(276).pdf
Pdf, 3.83 KB, 305×498 - Click the image to get file
Lawvere_F._W.,_Rosebrugh_R._-_Sets_for_Mathematics(2003)(276).pdf
>>56612
Вот эта книжка. Сам по ней занимаюсь.
>> No.56616 Reply
>>56615
Спасибо
>> No.56617 Reply
File: задача.jpg
Jpg, 131.03 KB, 1024×683 - Click the image to expand
edit Find source with google Find source with iqdb
задача.jpg
Аноны, помогите пожалуйста с комбинаторикой. У меня близится диплом, но висит долг еще за второй курс по этой не очень любимой мне дисциплине. Если не сдам - мне край. Препод выдает с ухмылкой задачи из книжки для 10-11 классов школы, но я даже их не знаю как делать - комбинаторика для меня - темный лес. В общем подскажите хотя бы идею решения пикрелейтед, по сути задача из теории графов, но я вообще не знаю, как ее делать. Препод говорил что-то про обход в глубину и в ширину, но я не понимаю, как метод разметки графа может помочь при ее решении.
>> No.56620 Reply
>>56617
К сожалению, не понимаю условие.
>> No.56623 Reply
File: Фото0182.jpg
Jpg, 779.58 KB, 2048×1536 - Click the image to expand
edit Find source with google Find source with iqdb
Фото0182.jpg
>>56620
В общем нужно написать прогу, которая бы эту задачку решала. Что мы имеем на вводе: клетки шахматной доски, занятые некими белыми фигурами, без разницы какими, но не ладьями. Также на ввод подается то самое "некоторое количество ходов", за которые ладьи должны будут побить черную фигуру, поставленную на любую свободную клетку. То есть практически белые фигуры занимают клетки, через которые не может бить ладья. Нужно найти количество ладей, которые необходимо, чтобы побить ту самую черную фигуру за заданное количество ходов. Я набросал на бумажке пикрелейтед, слева - та самая шахматная доска, черные точки на ней - это наши белые фигуры. Справа - граф, который получается из этой доски, вершины графа - это свободные клетки, а ребра - это те маршруты, по которым могут двигаться ладьи. В общем я вообще не представляю, как это все решить, был бы алгоритм, запрограммировал бы, но я даже понятия не имею, в чем тут хитрость.
>> No.56624 Reply
>>56623
Если нет никаких ограничений на клетки, которые могут занимать другие белые фигуры и пешки, то задачи неразрешима. Можно расположить так, что любым числом ладей чёрную фигуру бить не получится.
>> No.56625 Reply
>>56624
Почему же неразрешима? Ты говоришь о случае, когда черную фигуру побить будет невозможно - такое может быть, если только она окружена другими черными фигурами, то есть граф будет состоять из более чем одной компоненты связности, значит нужно будет ещё и сделать проверку на это, чтобы прога в случае чего выдавала ответ, например "побить черную фигуру невозможно". Но как вообще реализовать нахождение этого наименьшего числа ладей - вот это загадка.
>> No.56627 Reply
File: fig1.PNG
Png, 8.34 KB, 436×424 - Click the image to expand
edit Find source with google Find source with iqdb
fig1.PNG
>>56625 Я - не >>56624-кун, но по поводу задачи выскажусь. Его утверждение вполне верно, если предположить, что белые фигуры, помимо ладий, двигать нельзя, и они выполняют роль просто "помех на пути", то задача сводится к поиску связных компонент графа.
На пикрилейтеде серые клетки обозначают поля доски. Понятно, что если двигать их нельзя, то в клетки а7, а8 и b8 ладье не попасть из, допустим, а1. Так что нам потребуется как минимум две ладьи. Если же черная фигура окажется в h8, то её вообще нельзя будет взять ладьей. Для создания аналогичной "недоступной" фигуры для пункта б) задачи достаточно и одной белой фигуры (что продемонстрировано на примере g2 и h1). Количество связных областей на доске можно определить, например, волновым алгоритмом (это я как мимо-быдлокодер говорю, наверняка есть более изящные методы).
>> No.56629 Reply
>>56624
Отозван: в закрытые клетки можно поставить ладью тогда она займет место.

Подумал: графы для этой задачи не годятся.
Вопрос: Решение с использованием массива в 512 байт (64 строки длиной 64 бита) подойдет?
>> No.56630 Reply
>>56627
Ты говоришь всё верно, но есть одно НО: количество ходов, за которое должна быть взята черная фигура тоже задается, иначе задача конечно же была бы примитивынм поиском компонент связности. То есть даже на твоем пикрелейтед, если нужно, например, побить черную фигуру за 1 ход, то понадобится 7-8 ладей, сколько именно - не знаю, а вот уже для 2 ходов вполне хватит и 2 ладей. В том-то и сложность, что нужно определить число ладей для конкретного числа числа ходов, которое задается на вводе.
>> No.56631 Reply
>>56629
Сойдет любой алгоритм, препод говорит, что мы все говно, а он гуру, поэтому он не смотрит на вычислительную сложность алгоритма и скорость его работы, главное, чтобы вообще работал.
>> No.56633 Reply
>>56627
Кстати да, в изолированную клетку можно либо воткнуть ладью, либо решения не будет. Раз условием это не предусмотрено - не думаю, что будут такие тесты при сдаче. Та что наверное нужно рассматривать нормальную ситуацию, без всяких уловок.
>> No.56635 Reply
>>56631
It's kind of bruteforce.
Нет, я передумал. Вот, что нужно: 64-битные числа, описывающие характеристическую функцию множеств клеток на шахматной доске. Нужно уметь брать поразрядно складывать (брать объединения) в F_2. Также надо уметь брать мощности (число единиц в двоичном разложении).
Первое, что дается — это множества заполненных клеток.
Второе: число шагов.
Предлагаю: Создать массив, в котором каждая стока будет соответствовать свободной клетке, а ее содержимое будет показывать все клетки, которые можно бить, сходив один раз.
Это делается просто: берется клетка и в строку вписываются все клетки горизонтали (т.е. на соответствующие места ставятся 1) и вертикали до первой занятой клетки.
Потом исходя из числа возможных шагов вычисляется множество клеток, которые можно бить ладьей из неё. Пусть мы считаем для клетки x. Множество соединенных с ней клеток обозначим X. Для этого берется еще один массив, строка-множество X' для клетки x которого даются следующим образом:
1. X':=X
2. y∈X', тогда X':=Y⋃X'
3. Повторить пункт данное число шагов.
Потом нужно вычислить мощности X' для любого x и отсортировать их. Получится «рейтинг» клеток.
Теперь нам понадобится ещё одно число-множество P — множество клеток, которые займут ладьи.
Мы берем произвольную клетку (т.е. по порядку проходим) a и выбираем в рейтинге наивысшую клетку, для которой a∈X'. Саму x заносим в P. Пройдя по всем клеткам, вычисляем мощность P.
Конец.
>> No.56637 Reply
>>56635
> складывать
Глупость сказал. Тогда это симметрическая разность будет.
> Для этого берется еще один массив, строка-множество X' для клетки x которого даются следующим образом:
даётся
> Повторить пункт данное число шагов.
пункт 2. Очень неэкономно, кстати.
Замечание: x∈X.
>> No.56645 Reply
File: Доска.JPG
Jpg, 11.82 KB, 943×306 - Click the image to expand
edit Find source with google Find source with iqdb
Доска.JPG
>>56635
Клетка с наибольшим рейтингом не всегда будет лучшей. Например возьмем пикрелейтед - доска 3*3, белые кружки - изначально расставленные белые фигуры, зеленый кружок - та самая клетка с наивысшим рейтингом, но для того, чтобы побить черную фигуру за 1 ход, не нужно ее использовать, нужно поставить ладьи на место двух красных кружков. Да и если честно, я вообще мало что понял из написанного, вся эта задача делает меня грустить, тем более она точно должна решаться с помощью теории графов, она относится к разделу "Структуры данных: очереди, стеки, списки".
>> No.56646 Reply
А у меня такая проблема.
Я физик-теоретик-второкур, желаю изучить дифф. геометрию (скорее, систематизировать обрывочные знания).

У нас на физ. факе сей чудный предмет не читается (ибо, почему-то наше руководство считает, что все что нам нужно, нам расскажут преподы по физике, делая математические отступления).

Пробовал ходить на нее к математикам, но там какая-то хуета (пол семестра они решают простенькие задачки на кривые, касательные и т. п., а тензора на многообразиях не изучают вовсе).

Поэтому, прошу посоветовать годную книжку. Есть кое-какие пожелания. Я всей душой люблю формализмы. И мне хотелось бы изучать все в следующем порядке:

1) определение топологического и метрического пространств, простейшие теоремы
2) определение геоморфизма, n-мерного многообразия, гладкого многообразия
3) изучение гладких многообразий (абстрактных и конкретных), примеры, задачки.
4) тензора на многообразиях, связность, ковариантное дифференцирование, тензор кривизны и т. п.

Есть ли чудная книжка, следующая примерно такой логике (ибо такой порядок изложения кажется мне наиболее последовательным). В книжках по дифф. геометрии, которые я встречал, почему-то совсем нет основ топологии, которые необходимы, а мне хочется понимать все очень системно.

В наличии имеются твердые знания линала, матана, тфкп (хоть оно и не особо нужно здесь), мат. логики (на уровне спецкурса для физиков).

Будьте няшами!
>> No.56650 Reply
Здравствуйте. Не подскажете, где можно почитать про гиперплоскости? Способы задания, типовые задачи и всё в таком духе.
>> No.56695 Reply
>>56608
>>56615
Думаю, перед математикой нужно позаниматься метаматематикой.
>> No.56699 Reply
>>56645
Действительно, есть проблема, но из картинки ответ следует. Не сверху вниз, а снизу вверх.
Стоишь обратный рейтинг - ищешь пересечения. Тогда проблемные точки будут закрыты наименьшим числом ладей.
>> No.56702 Reply
>>56646
Дубровин-Новиков-Фоменко - твое счастье. Правда, на него многие катят бочку - типа устарел и слишком там все неинвариантно изложено, но раз ты теорфизик, тебе координаты будут ох как нужны. Еще можешь глянуть сюда http://ium.mccme.ru/s10/difgem.html, записей лекций там нет, но есть видеозаписи, и очень годные листки. Правда, предполагается, что чел уже знает общую топологию, когда слушает этот курс. Алсо, вначале там тоже
> какая-то хуета, простенькие задачки на кривые, касательные и т. п.
но они используются как мотивировка к введению всяких касательных расслоений, связностей и прочего.
>> No.56705 Reply
>>56568
Начал я кострикина читать, даже стыдно говорить, что ничего понять не могу.
>> No.56708 Reply
>>56695
Не нужно. Весьма бесполезное занятие, если не собираешься быть логиком и доказывать специфические факты о формальных системах. Та книжка не требует специальных знаний.
>>56646
Сарданашвили — физик. Написал пять книжек «Современные методы теории поля», куда вошли элементарная топология, дифференциальная геометрия, теория Морса и т.д. Даже многообразия струй есть. Всё, что тебе надо, имеется там. Доказательств и теорем особо нет, но они тебе и не нужны. А так, русские книжки, как правило, глупые. S.Ramanan "Global calculus" - хорошая книжка, но топологию да алгебру надо знать. Если будешь по пути математика идти, её читай.
>>56705
На какой странице не понятно? Винберг лучше же.
>> No.56715 Reply
Аноны, кто изучал дифгем с дифтопологией самостоятельно? И за какой срок хотелось бы знать.
>> No.56716 Reply
>>56646
Бро, тебе нужно ориентироваться совсем не на это. Тебя, например, топологическая теория поля вряд ли интересует, намного важней, как замешать взаимодействие материи в данную модель. Поэтому гугли книжечку по следующим моментам:
1)>тензора на многообразиях, связность, ковариантное дифференцирование, тензор кривизны и т. п.
это гравитация и очень просто. это должно быть максимум на 150 страниц. Понимай, чьи индексы верхние, чьи нижние - кто за что отвечает. Тензор энергии-импульса.
+тетрадный формализм
2) теория поля.
Грубо говоря, в гравитации тоже есть аналог связности на касательных расслоения (какой?), но тебе нужно именно понимать, как живут формы (тензоры) на расслоениях, как они связаны между собой.
3) топологические теории.
тут тривиально все. как только поймешь, кто такие солитоны - все просто.
4) обязательно пойми, кто такие спиноры. Потому что бозонные теории крайне просты, а вот спиноры - не все физики даже знают, что это. Максимум "то, что в уравнении Дирака".

Вообще, если ты не твердотельщик, то обязательно учи: калибровочные теории и группы Ли; гравитация; спиноры; стандартная модель; +(струны и суперсимметрии);

Новикова не читай, нмушников тоже не читай. Там очень долго основные моменты разбираются, так до хоть сколько-нибудь интересной теории не дойдешь. Возьми за образец двухтомник по теории суперструн виттена. А еще лучше поставь себе цель, например, разобраться в стандартной модели или в квантовой теории поля. Вообще, русские книжки желательно не читать. Хотя есть одно исключение, правда, я забыл совсем название- но тебе бы точно подошло. Что-то типа "Дифференциальная геометрия для физиков". Еще советую Gravitation and gauge symmetries - Milutin Blagojević. Вообще, читай книжки для теор. физиков. А то с математиками хуй куда продвинешься: "а ты физик! тогда читай книжку с координатным подходом."

И еще раз, лучше ставь цель - разобраться в модели. А не заботать диф. гем. Привет!
>> No.56719 Reply
>>56716
> Там очень долго основные моменты разбираются
Даже не основные, а суперпримитивные. А обрамляется помпезно, с нагромождениями.

>>56715
Да, я другой кун. Поэтому вопрос ещё в силе.
>> No.56722 Reply
>>56716
> как живут формы (тензоры)
Будто бы это одно и то же.
>> No.56734 Reply
>>56650
Хотя бы авторов учебников назовите, пожалуйста.
>> No.56735 Reply
>>56646
Тайманов - Лекции по дифференциальной геометрии. Это самые основы, определения и тд.
Далее, тебе надо в сабж, именующийся "Риманова геометрия". Тут уже надо глядеть на общую подготовку. Как у тебя с топологией и матаном/функаном?
Фоменко, кстати, рекомендую почитать на досуге. Всё таки книга интересная, хотя далеко не самая лучшая.
>>56650
Да в любом учебнике по функану, я думаю. А что конкретно надо?
>>56708
> А так, русские книжки, как правило, глупые
Да их не так уж и много. Если сюда не включаются переводные. А вообще да, если серьёзно заниматься геометрией/топологией, то надо привыкать к английским терминам, ибо чем дальше в лес - тем меньше русских статей и книг.
>>56715
Я изучал и продолжаю изучать алгебраическую топологию в течение 3 лет. Могу смело заверить - я её не знаю. Так что тут зависит от того, какими мерками мерить.
>>56716
> ты физик! тогда читай книжку с координатным подходом
Точно. --ironymod=on Читайте своего Рашевского и отъебитесь нахуй, математики тут делом заняты.
> нмушников тоже не читай
Что за нмушники? Вербицкий и ко?
>>56722
Первое - частный случай второго же. Формы физикам оче полезны.
>> No.56740 Reply
>>56735
> Я изучал и продолжаю изучать алгебраическую топологию в течение 3 лет. Могу смело заверить - я её не знаю. Так что тут зависит от того, какими мерками мерить.
Какой уровень? В ВУЗе учился?
Собсна, ещё бы ответов для статистики.
> Точно. --ironymod=on Читайте своего Рашевского и отъебитесь нахуй
Тред математики, вопросы по математике, всё пучком вроде бы.
>> No.56761 Reply
>>56708
> Не нужно. Весьма бесполезное занятие, если не собираешься быть логиком и доказывать специфические факты о формальных системах.
Собираюсь. Думаю, любому математику стоит обратить немного внимания на основания.
> Та книжка не требует специальных знаний.
Заметил.
56695-кун
>> No.56768 Reply
С недосыпа запостил в тонущий тред, продублирую тут.
Матанач, посоветуй, позязя, какую-нибудь внятную книжку по теории устойчивости динамических систем.
>> No.56770 Reply
>>56768
А впрочем, буду более прямолинеен. Есть у меня, например, динамическая система вида http://mathbin.net/92573 , и я, например, нашёл особые точки, приравняв правые части уравнений нулю. Хочу исследовать характер этих точек (понять, что это за точки - центр, фокус, седло, etc). Задачник Филиппова (равно как и учебник Треногина) в один голос говорят, что нужно линеаризовать систему, разложив нелинейную часть по Тейлору в окрестности этой точки и взяв члены разложения первого порядка. Но у меня нелинейные куски - квадраты x и y и, что наиболее страшно, произведение x на y, а их как по Тейлору не раскладывай, получатся те же квадраты и произведение. Где я неправильный?
>> No.56776 Reply
>>56770
Бака я, бака, сейчас всё подробно расписал и получилось:
http://mathbin.net/92586 . Всё правильно сделал?
Да, да, сам шучу, сам смеюсь.
>> No.56777 Reply
>>56735
> А что конкретно надо?
Хотелось бы название и авторов учебника, десу.
>> No.56778 Reply
>>56735
> Первое - частный случай второго же.
Да, а огрызок - частный случай яблока фрукта.
Внешняя форма на многообразии - это дважды частный случай тензорного поля.
>> No.56780 Reply
>>56778
О чем ты споришь?
>> No.56782 Reply
>>56780
О том, что разобраться в формах не то же самое, что разобраться в тензорных полях.
>> No.56783 Reply
>>56782
Ок. Поспорили.
>> No.56784 Reply
>>56783
Ты скучный.
>> No.56787 Reply
>>56777
Например, Кутателадзе - Основы функционального анализа. с.50, там же основное утверждение про гиперплоскости и теорема отделимости. Но это чисто знакомство с функановской точки зрения. Тебе надо знакомство или плотное изучение? Комбинаторика, топология гиперплоскостей, да? Тогда это совсем другой разговор, рекомендую книгу:
Orlik, Peter; Terao, Hiroaki - Arrangements of Hyperplanes.
>> No.56789 Reply
>>56787
> Кутателадзе - Основы функционального анализа
Феерически бредовая книжка. Сразу видно: автор не математик.
>> No.56798 Reply
>>56541
Спасибо большое!
Загружу, почитаю. Задачник Демидовича помню - единственный задачник, не вызывавший у меня ступора.
>> No.56804 Reply
>>56789
> Феерически бредовая книжка.
Чем подкрепишь?
>> No.56808 Reply
>>56515
Ты еще тут? Кун, который решает задачу Римана аналитически?
Я сейчас сел писать точное решение. Пока что написал расчет P и U в области между волнами. Щас буду обмозговывать как определять, какие там волны идут и как считать параметры сплошной среды около фронта, а для волны разрежения еще и надо въехать, как считать параметры в ее области движения.
>> No.56813 Reply
>>56804
1. Автора тянет на фильтры. Он строит ими топологию. Такого зверства даже в энциклопедии общей топологии не видел, которая является, видимо, большим авторитетом.
2. Автор сильно использует общую топологию, при этом знает о решетках, но не знает, что по их вине общая топология уже умерла. Как ни странно, в книжке для advanced undergraduate students умных школьников и первокурсников 2006 года есть указание на это печальное событие.
3. Автор не использовал теорию категорий, поэтому, кажется, и не знал о смерти общей топологии. Но коммутативные диаграммы есть. Да и книжки с категориями в списке использованной литературы присутствуют.
4. Автор напридумывал обозначений и переписал определения (не в лучшем виде). Много места отведено рассуждениям "не вполне корректно писать множество X вместо четверки (X,Y,Z,T)", что лишне.
5. Введенные понятия не мотивировались ничем.
6. Книга не такая плохая, как мне показалось с первого взгляда, но учится по ней не стоит, а вот изучать архаичную дребедень — пожалуйста. Почему-то пучки были, но в такой книжке они не к месту.
Про гиперплоскости, раз кто-то интересуется. Здесь, например, вводит "гиперподпространство", которое встречается в одной статье на arxiv.org совершенно в ином смысле.
>> No.56816 Reply
>>56813
> в книжке для advanced undergraduate students умных школьников и первокурсников 2006 года есть указание на это печальное событие.
Что за книжка?
>> No.56820 Reply
>>56816
Topology and groupoids, Brown.
>> No.56821 Reply
File: 246334.jpg
Jpg, 21.65 KB, 300×220 - Click the image to expand
edit Find source with google Find source with iqdb
246334.jpg
>>56787
> Кутателадзе - Основы функционального анализа
Это кто тут такой нерадивый, что вспомнил про эту кошмарную книжку? Доказательства там разобрать некоторые в принципе невозможно, не говоря уже о костном языке и устаревшей информации, хотя для основ функана на самом деле пойдёт. И в задании топологии через фильтры не вижу ничего плохого. Но в целом книжка фу, если нужен учебник по функану, то лучше старого доброго Колмогорова/Фомина читать или Рудина.
> что по их вине общая топология уже умерла
Общая топология умерла скорее от того, что в ней нет задач, которые имело бы смысл решать. Такая же застывшая наука, как теория чисел, например.
> Автор не использовал теорию категорий, поэтому, кажется, и не знал о смерти общей топологии.
Алё, это книга по основам функана, а функан у студентов ассоциируется в первую очередь с операторами и банаховыми пространствами, нужны категории - обратитесь к маклейну или моей любимой The Joy of Cats.
> Книга не такая плохая, как мне показалось с первого взгляда, но учится по ней не стоит, а вот изучать архаичную дребедень — пожалуйста.
Да на русском все такие. По функану по крайней мере. Ну я только Хелемского более менее выделю.
>> No.56822 Reply
>>56821
> лучше старого доброго Колмогорова/Фомина
Устарел он очень уж.
> И в задании топологии через фильтры не вижу ничего плохого.
А я вижу: бессмысленная трата времени. Они были были в моде во времена Бурбаки - оказались не удобными. Вот и всё.
> банаховыми пространствами
А нужно с категориями банаховых пространств.
> The Joy of Cats.
Хорошая такая книжка. Мне тоже нравится.
> Ну я только Хелемского более менее выделю.
Вот-вот, чудная книжка с категориями. Все собираюсь бумажную версию заказать.
>> No.56823 Reply
>>56822
неудобными
Фикс
>> No.56853 Reply
File: волны разрежения.jpg
Jpg, 19.51 KB, 561×420 - Click the image to expand
edit Find source with google Find source with iqdb
волны разрежения.jpg
>>56479
Оно?
>> No.56871 Reply
Вейвлетобоги есть в треде?
>> No.56884 Reply
>>56853
Какие начальные условия были? Сколько ячеек? Почему решение задавалось на участке [0;100] (для теста должно задаваться [0;1])?
>> No.56895 Reply
>>56884
участок [0,1], просто 100 точек, не подписал.
p1 = 1; %%давление
ro1 = 0.2; %%плотность
u1 = 0.8; %%скорость

p2 = 2;
ro2 = 1.3;
u2 = 1.9;

Волны
D1 = -2.0868
D1_asterisk = -1.9793

D2 = 3.5013
D2_asterisk = 3.2421
поэтому участки расплыва между двумя фронтами не видны почти. Если найдешь другие тестовые условия говори. Но главное, что давление в области между волнами упало. Щас буду метод Годунова делать.
>> No.56896 Reply
File: волны разрежения2.jpg
Jpg, 18.87 KB, 561×420 - Click the image to expand
edit Find source with google Find source with iqdb
волны разрежения2.jpg
>>56884
dx = 0.01;
dt =0.001;
Вообще, да условия надо будет подобрать нормально. А то нихуя не видно.
>> No.56911 Reply
Анон, что такое - "комбинир. метод" в интегрировании? Мне вернули сегодня типовик с этой ерундой, мол перерешать этим методом, я гуглю, но не могу найти ответ.
>> No.56938 Reply
File: 3e6f47c5922e55104eb138865999a067.png
Png, 4418.79 KB, 1365×1355 - Click the image to expand
edit Find source with google Find source with iqdb
3e6f47c5922e55104eb138865999a067.png
>>56896
Нужно поставить 500 ячеек и условия такие:

Слева (ячейки от 0 до 249):
pressure = 1.0
density = 1.0
speed = 0.0

Справа (ячейки от 250 до 499):
pressure = 1.0
density = 0.0
speed = 0.0

Адиабата (gamma): 5/3
dx = 1 / 500 = 0.002
dt = 0.001

Сами мы тоже скоро допилим свою версию (вернее, препод нам поможет, но он немного ленивый ;__; )
Однако все равно надо сверяться будет. И с вашими результатами тоже.
>> No.57014 Reply
>>56708
Да ВСЕ непонятно, унитарное, евклидовы пространства. Ортонормированные базисы - как будто я в ОМСК попал.
>> No.57063 Reply
File: 1330977422003.jpg
Jpg, 51.57 KB, 594×476 - Click the image to expand
edit Find source with google Find source with iqdb
1330977422003.jpg
>>57014
Бро, я сейчас тоже линейную алгебру покуриваю, буду рад обсудить что-то конкретное.
У меня тем временем есть вопрос к доброанону, буду признателен за ответ.
Вот смотри. Есть самосопряженный линейный оператор А в унитарном пространстве U. Допустим, мы установили факт, что если построить оргтогональное дополнение к какому-то его собственному вектору, то это будет подпространство, инвариантное относительно А, обозначим его U'
Теперь рассмотрим сужение A на U'. Ортогональное дополнение к любому его собственному вектору из U' будет также инвариантно относительно A. Ну и так далее, пока мы не построим ортонормированный базис из собственных векторов.
Вопрос вот в чем - на основании чего мы можем утверждать, что нужное для этого количество собственных векторов у такого оператора вообще есть, т.е. на каждом шагу в ортогональном дополнении обязательно найдется собственновектор?
>> No.57080 Reply
>>57063
Число собственных векторов равно числу собственных чисел равно размерности оператора.
>> No.57081 Reply
>>57080
> Число собственных векторов равно числу собственных чисел
Уверен?
>> No.57082 Reply
>>57081
Да, там однозначное соответствие же, а пространство таки унитарное.
>> No.57109 Reply
File: иконокот.jpg
Jpg, 52.89 KB, 549×481 - Click the image to expand
edit Find source with google Find source with iqdb
иконокот.jpg
>>57080
Я не уверен, что понял утверждение.
Размерность оператора - это его ранг?
>> No.57178 Reply
File: P220412_20.29.jpg
Jpg, 323.64 KB, 2048×1536 - Click the image to expand
edit Find source with google Find source with iqdb
P220412_20.29.jpg
Анон, как решить 5 уравнение?
Я пробовал разбить 6х^2=4x^2+2x^2, но замена не выходит.Ответ 1.
>> No.57186 Reply
File: Формальные сис......
Png, 130.08 KB, 1789×890
edit Find source with google Find source with iqdb
Формальные сис....png
File: 1274863244467.jpg
Jpg, 691.20 KB, 1104×1168
edit Find source with google Find source with iqdb
1274863244467.jpg

>>57178
Погугли "делить уравнения столбиком"

Требуются заинтересованные добровольцы
1)Гляньте пикрилейтед
2) Немного обо мне
Я студентота 3лвла физ специальности. К текущему моменту я изучил, сдал, забыл дикое кол-во матдисциплин и, внезапно, мне они все понадобились. Да-да я занимаюсь квантовой теорией поля. Я понял, что необходимо выстроить в голове хотя бы приблизительную схему математики. Недавно я встретил англоязычный пикрилейтед и задумал вот что: я хотел бы делать и слушать научпоп математику. Пример: схему на пикрилейтеде можно рассказать за 5-6 минут так, чтобы примерно понимать что есть в математике. Каждый пункт в схеме так же можно общо рассказать за 2-3 минуты.
3)Что это даст? Это даст интуитивное широкое понимание математики(а зачастую больше и не надо). Причем как авторам, так и слушателям.
4) Форма. Надо, однако, уходить от формата текста я щитаю и переходить к презентациям, для таких "кругозорных" целей это гораздо понятней.
5) В одиночку я не справлюсь с такой задачей. Слишком многого я не знаю даже примерно, кроме будет полезно много обсуждать материал.

Если бы нашлось 4-5 людей которые готовы этим заниматься ради самообразования нужно было бы скооперироваться.
Предполагаемый(мной) формат примерно таков: кто-то делает сценарий презентации, идет обсуждение сценария и собственно делаем презентацию(т.е. видеоряд +голос). Это добавит еще один мотивационный пункт: "несение света в массы". Кроме того взгляд со стороны полезен.
В этом "курсе" не предполагается доказательств, он исключительно обзорен. Кроме того в титрах презентаций нужно давать ссылки на учебники.
Желающие заниматься отпишитесь итт.

В конце концов тред называется "кафедра математики", а не сделай за меня домашку по алгебре.
>> No.57187 Reply
>>57186
Это все далеко от квантовой теории поля же. Лучше Боголюбова осиль, как мне кажется. В любом случае, я пас.
>> No.57191 Reply
>>57187
Как раз боголюбова я сейчас и читаю, и даже математику понимаю, но проблема не в этом: я не чувствую фундамента. Не чувствую что откуда идет, а впереди всякие гравитации с топологиями. Цель не столько мне с ктп разобраться, сколько основу сделать.
>> No.57192 Reply
>>57191
Множество.
Кортеж.
Прямое произведение множеств.
Отношение.
Отображение.
Операция.
Универсальная алгебра.
Алгебраическая структура.

Магма.
Квазигруппа.
Петля.
Полугруппа.
Моноид.
Группа.
Абелева группа.

Кольцо.
Тело.
Поле.
Векторное пространство.
Метрическое векторное пространство.
Евклидово пространство.
Аффинное пространство.

Модуль над кольцом.
Алгебра над кольцом.

Ну и так далее.
>> No.57194 Reply
>>57191
> и даже ... понимаю
> я не чувствую фундамента
Значит не понимаешь.
>> No.57196 Reply
>>57192
И что? Еще раз, мне нужно изложение от общего к частному(сначала общая картина, потом все более мелкие детали). Я могу взять книжки по соответствующим направлениям и просто закопаться в них(я так и сделал собственно)

>>57194
На текущий момент там нету ничего сложнее свертки по индексам и производных, что я там не понимаю по-твоему? Чес-слово ощущение, что ты просто поумничать решил, извини коль что не так.
>> No.57199 Reply
>>57186
> Формальные сис......
Нет. Не отражает ни устаревшего строения математики, ни текущего. Текущее строение тоже я не знаю, ибо кроме определения бикатегории ничего не известно мне из таких понятий. Учебник на 950 страниц, и пока не до фундамента.
>>57192-кун ближе к истине, но у него терминология кривая и куча ненужных вещей.
>> No.57201 Reply
>>57196
> На текущий момент там нету ничего сложнее свертки по индексам и производных, что я там не понимаю по-твоему?
Взаимосвязей с фундаментом? Концепт тебе ясен, но понимаешь ли ты, откуда это взялось и что отражает?
>> No.57202 Reply
>>57201
Для понимания линала нужно ли знать докво теоремы Геделя? Ну в общем, наверное, да. Все-таки она лежит на уровень ниже.
В общем мы говорили о разных уровнях понимания и я тебя не понял. Впрочем понимаешь ли ты хоть что-нибудь с твоим определением понимания?

>>57199
> Нет. Не отражает ни устаревшего строения математики, ни текущего.
отлично, как раз для конструктивной критики мне и нужны коллеги.
>> No.57204 Reply
Анон, что такое ядро и образ оператора, нифига не могу понять билет
>> No.57210 Reply
>>57199
Почему ж ненужных? Одно цепляется за другое, последующее за предыдущее.
Прямое произведение - всевозможные кортежи, отношение - некоторое подмножество прямого произведения; группа - петля-моноид и т.д.
>> No.57211 Reply
>>57202
> Впрочем понимаешь ли ты хоть что-нибудь с твоим определением понимания?
Да. И не в теореме Гёделя дело.
>> No.57212 Reply
File: Screenshot-Yojouhan Shinwa Taikei - 04 RAW (CX 848x480 H.264 AAC).mp4-7.png
Png, 41.53 KB, 848×480 - Click the image to expand
edit Find source with google Find source with iqdb
Screenshot-Yojouhan Shinwa Taikei - 04 RAW (CX 848x480 H.264 AAC).mp4-7.png
>>57204
Пусть А- оператор, тогда множество всех векторов х, таких что
Ах=0 называется ядром.
Или кратко: ker A ={x: Ax=0}

Пример: есть оператор проекции из 3д на плоскость ху. Тогда ядром оператора будет множество всех векторов у которых только z-довая компонента. В частности вектор (0,0,1) находится в ядре.

Образ - множество значений оператора.
Или кратко
Im A = {y: E x, y=Ax}

Пример образом оператора проекции является множество всех векторов в плоскости ху: для любого такого вектора х существует вектор в 3-хмерном пр-ве у(хотя бы он сам) который при проекции переходит в х.
На крайний случай, есть двухмерное пространство и трехмерное. И даже если вектор как тебе кажется один и тот же, это разные объекты. Условно в 2д это набор чисел (1,1), а в 3д это (1,1,0). И оператор проекции переводит трехмерный вектор в двухмерный.
>> No.57213 Reply
>>57202
> отлично, как раз для конструктивной критики мне и нужны коллеги.
> для конструктивной критики
Много хочешь.
Вот следующие замечание: из исчисления предикатов первого порядка следует только какая-нибудь теория множеств (с классами или без). Для понимания теорий первого порядка знание булевых алгебр не нужно (это просто модель пропозиционального исчисления). Формальная арифметика и те натуральные числа, что в теории множеств, разные. Если посмотреть на схему индукции в формальной арифметике, то можно заметить, что она сформулирована лишь для счетного числа свойств, когда для обычной арифметики их континуум (такую структуру натуральных чисел легко формулируют в теории множеств).
То есть:
исчисление предикатов первого порядка -> теория множеств -> и т.д.
Формальная арифметика и теории групп (как теория первого порядка) нужны только логиками и не предоставляют рабочих объектов и теорем. Есть топология ещё. Почему-то про нее забыли.
>>57204
Запомни: ядро - то, что уходит в нуль (общепринятая вещь). Образ - это образ (множество всех значений отображения).
>>57210
> Петля.
> Универсальная алгебра.
> Алгебраическая структура.
Ну вот я не знаю, где работают с этими вещами. Универсальные алгебры - не знаю кому они нужны кроме тех, кто ими, собственно, зачем-то занимается. Задача математика не есть назвать большее количество понятий новыми терминами.
Вот, кстати,
> Векторное пространство.
есть, а модули потом. Нехорошо.
>> No.57216 Reply
>>57204
F:A->B
образ - Image - F(A) это обычный теоретико-множественный образ
ядро - kernel - Ker(F) это все элементы из A переходящие в 0 в В. Не трудно проверить что ядро является подмодулем.
>> No.57217 Reply
>>57212
Спасибо, только не понял
Что в обрайзе значит y: - игрик, такое что суще-т икс,у =AX или что?
>> No.57222 Reply
File: Screenshot-Yojouhan Shinwa Taikei - 02 RAW (CX 848x480 H.264 AAC).mp4-1.png
Png, 221.61 KB, 848×480 - Click the image to expand
edit Find source with google Find source with iqdb
Screenshot-Yojouhan Shinwa Taikei - 02 RAW (CX 848x480 H.264 AAC).mp4-1.png
>>57213
> про схему
Вообще-то я тупо попереводил английский аналог с форчана(и сделал это явно плохо), каюсь.
>>57217
У меня ошибка в логике. Прости.
Im A= {y: y=Ax}
Суть ошибки вот в чем. Я не потребовал, чтобы у любога икса Ах лежал в образе, а потребовал только что бы у любого элемента образа был прообраз. Этого мало.

Итак попробую не зафейлить второй раз:
Для оператора А,
назовем образом элемента х, элемент у =Ах.
Тогда образом оператора А называется множество всех образов х. Т.е. мы берем все х из соответствующего пр-ва, действуем оператором, результат(все множество) называем образом оператора.
>> No.57223 Reply
>>57213
> Ну вот я не знаю, где работают с этими вещами.
Я их использовал только как промежуточную ступеньку между множеством и группой. По-моему, так красивее.
> Универсальные алгебры - не знаю кому они нужны
Программерам же.
> Векторное пространство есть, а модули потом. Нехорошо.
Согласен, маху дал.
>> No.57224 Reply
>>57222
> Тогда образом оператора А называется множество всех образов х. Т.е. мы берем все х из соответствующего пр-ва, действуем оператором, результат(все множество) называем образом оператора.
Да
>> No.57232 Reply
>>57222
> аналог с форчана
> 1274863244467.jpg
Это что ли? Писали несколько странные люди. Забыли, кстати, соединить R^n c метрическими пространствами. В определении многообразия алгебраическая структура R^n не имеет никакого значения.

Перечитал твой текст (понял смысл). Сам собирался сделать что-то подобное, но без популярной математики и звука:
1. Неумение составлять картинки для TeX'а.
2. Полное введение исчисления предикатов первого порядка с теорией множеств утомительно и не мотивируется никак, следовательно, не подходит для обучения в первый раз.
3. Сам знаю недостаточно.
4. Сказывается болезнь.
А чтобы рассказывать интуитивно нужно обладать гораздо большим опытом и знаниями.
>>57223
> По-моему, так красивее.
Не стоит вводить понятие, чтобы один раз его использовать.
> Программерам же
http://dobrochan.ru/s/res/11962.xhtml №11997
Не ты ли удивил меня странной терминологией? Не писал в тот тред.
>> No.57234 Reply
>>57080
Немного подумал, и вот.
Бро, я не совсем про то спрашивал. Уточню вопрос. Как обосновать такое утверждение:
"у любого оператора есть по меньшей мере один собственновектор, и раз оргтогональное дополнение к нему - инвариантное подпространство, значит в нем есть по меньшей мере еще один собственновектор"
> Число собственных векторов равно числу собственных чисел
Это в общем случае произвольного самосопряженного оператора в унитарном пространстве неверно, я решительно не понимаю, о чем речь
>> No.57238 Reply
>>57232
> Не ты ли удивил меня странной терминологией?
Нет, не я. А что в ней странного?
> Не стоит вводить понятие, чтобы один раз его использовать.
Эти объекты полезны программистам и выглядят довольно логично, поэтому должны иметь название.
>> No.57256 Reply
>>57191
Гравитация - это очень просто же, дурачек. Ты же уже здесь писал, правда? Я тебе сказал еще Blagojevicа читать. Гравитация + тетрады, чтобы там фермионы были - это очень простой топик. Намного проще, чем Боголюбов. А если ты понимаешь Боголюбова, то ты ниибечески крут. Я там зависал над каждым уравнением.
>>57194
Доброчую господина.
>> No.57291 Reply
>>57178
Методом Феррари, очевидно.
>> No.57294 Reply
File: Screenshot-Yojouhan Shinwa Taikei - 04 RAW (CX 848x480 H.264 AAC).mp4-17.png
Png, 262.84 KB, 848×480 - Click the image to expand
edit Find source with google Find source with iqdb
Screenshot-Yojouhan Shinwa Taikei - 04 RAW (CX 848x480 H.264 AAC).mp4-17.png
>>57256
> дурачек
а разве не через о? не знаешь что ответить оппоненту - приебись к орфографии
> Ты же уже здесь писал, правда?
нет, первый раз итт
> Я там зависал над каждым уравнением.
Я не имею ввиду, что мне сходу понятны преобразования, я тоже "зависаю".

Ок, хрен с вами господа.
>> No.57313 Reply
>>57294
ага, через о.

Пролистай тред. Тут уже была поднята точно такая же тема.
>> No.57347 Reply
Аноны, анончики, в связи с моим временным уходом от криптографии в кодирование понадобилось резко вспоминать ТЧ. У нас когда-то на 1 курсе был сабж, но разумеется, я ничего из него не вспомню. Накидайте годных книжек, можно (даже желательно) на английском, ибо я после криптографии сильно к английским терминам привык. Нужен именно базовый курс, всякую муть про идеалы и прочее - не нужно.
>> No.57348 Reply
File: 01030.png
Png, 382.10 KB, 640×360 - Click the image to expand
edit Find source with google Find source with iqdb
01030.png
>>57347
G.H. Hardy, E.M. Wright: An Introduction to the Theory of Numbers
K. Ireland, M. Rosen: A Classical Introduction to Modern Number Theory
>> No.57482 Reply
File: IMG_20120424_121423.jpg
Jpg, 845.27 KB, 2592×1944
Your censorship settings forbid this file.
unrated
Анон, может быть, ты заскучал - тогда порешай, пожалуйста, эти задания, очень прошу. С меня няшки :3
>> No.57491 Reply
>>57482
Или хотя бы намекни.
>> No.57520 Reply
>>57482
Уже можешь не решать, поздно.
>> No.57538 Reply
File: Безымянный.jpg
Jpg, 15.73 KB, 748×160 - Click the image to expand
edit Find source with google Find source with iqdb
Безымянный.jpg
Нужно доказать, что пикрелейтед делится на 84. Судя по ответам, доказывается индукцией, но в ответах задача не разбирается. Уже час пытаюсь решить, выручай, анон.
>> No.57541 Reply
File: rod_xxvi__by_wen_jr-d4lbyoh.jpg
Jpg, 238.05 KB, 900×1674 - Click the image to expand
edit Find source with google Find source with iqdb
rod_xxvi__by_wen_jr-d4lbyoh.jpg
Доброго времени суток.

Пишу курсач по нейронным сетям, и интересует один очень специфический вопрос - какие есть способы точной (!) аппроксимации функции нейронной сетью? Или же работы, статьи на эту тему.

Идея курсовой элементарна, как синус - нужно представить в виде нейронной сети функцию x^2 (либо e^x). Но я не успеваю толком разобраться. Просто обучить каким-то алгоритмом не пойдёт, ибо я даже не знаю, сколько нейронов брать.

Если кто что знает, буду очень благодарен.
>> No.57573 Reply
File: 4 и две задачи 3.gif
Gif, 1618.86 KB, 2560×1440 - Click the image to expand
edit Find source with google Find source with iqdb
4 и две задачи 3.gif
>>57482
Если у тебя есть ответы, проверь пожалуйста, сам только прохожу эту тему.
>> No.57575 Reply
>>57573
В первом ты 4 в ответе потерял. Достаточно равенства нулю одного из слагаемых. И весь отрезок от -4 включительно до -5 не включая.
мимо
>> No.57578 Reply
>>57575
А во втором забыл, что нужна положительная дробь. Следовательно и числитель, и знаменатель должны быть положительны. То есть x >= 1 и x > 7, следовательно x можно брать с отрезка от 7 до бесконечности.
Либо и числитель, и знаменатель отрицательны. То есть x или от [-1,0], или ([0,1] в числителе и x от -2 до трех в знаменателе. Но пересечение этих отрезков пусто, так что остается только первый вариант.
>> No.57588 Reply
File: MizoreShirayukiSmallerKB1.jpg
Jpg, 257.79 KB, 1024×576 - Click the image to expand
edit Find source with google Find source with iqdb
MizoreShirayukiSmallerKB1.jpg
>>57575
Я нихуя не понял, но ладно, попробую разобраться.
>>57578
Да, понял косяк. Я подумал, что числитель при любых х > 0, что неверно.
Спасибо, держи няшу.
>> No.57602 Reply
>>57588
Ну смотри, у тебя в третьей строчке написано (4 - x) * log ... >= 0. А строкой ниже справо это раскрыто в (4 - x) < 0 и log ... < 0. Если в начальном неравенстве знак нестрогий, то и в получившихся равенство нулю возможно, нэ? Так что интервал от 4 включительно до бесконечности. Это первое.
А в столбике слева у тебя получился x на интервале от минус бесконечности до -4. -4 по тем же рассуждениям надо включить в интервал. Взять пересечение с областью определения(x > -5) и записать в ответ.
>> No.57628 Reply
>>57538
n=1 16-9-7=0
n=2 256-81-7=175-7=168=84*2

(a)По индукции пусть n=k+1, и доказано для k.
4^[2(k+1)]-3^[2(k+1)]-7=4^[2k]16-3^[2k]9-7=9(4^[2k]-3^[2k]-7)+4^[2k]*7+56

(b)Докажем быстренько по индукции, что и это делится на 84:
4^[2k]*7+56
k=1 112+56=168=84*2
k=m+1, доказано для m.
4^[2(m+1)]7+56=4^[2m]716+56=16(4^[2m]7+56)-5615=16(4^[2m]7+56)-783*5
Первая часть делится по индукции, правая часть делится на 84, поскольку 84=422=214=734 очевидно является делимым.

Таким образом, мы индукционно доказали, что при любом k правая скобка выражения (a) делится на 84. При этом левая скобка делится по индукции. Следовательно, делится всё выражение, и задача решена.
>> No.57629 Reply
>>57628
> Первая часть делится по индукции...
Левая, конечно же.
>> No.57634 Reply
>>57628
Я немного по-другому доказал: для n=k+1 будет в итоге 9(4^2k - 3^2k -7) +7(4^2k + 8)
А поскольку 7(4^2k +8) делится на 7, 4 и 3(т.к. 4^2k при делении на 3 дает в остатке 1), то и само это число делится на 84.
>> No.57642 Reply
>>57578
А как же метод интервалов? Просто разложить логарифмы в числителе и знаменателе по формуле, найти корни и решить :3
Ту контрольную я зафейлил конкретно.

>>57573
Анон, а откуда у тебя такие же задания? Ты не из лицея?
>> No.57644 Reply
>>56938
А можно какие-нибудь физические условия поставить? Потому что давление 1 при плотности 0 - это пиздец какой-то. А вообще, сегодня запилю метод Годунова и посмотрю, насколько у меня неправильно все.

P =

   NaN
U =
   NaN
D1 =
   -1.6330
D1_asterisk =
   NaN
D2 =
   Inf
D2_asterisk =
   NaN
>> No.57671 Reply
>>57644
Нашел ошибку в своей проге.
В твоем случае, влево идет ударная волна со скоростью 1.2910, на ней разрыв плотности. Все остальное по нулям.
>> No.57734 Reply
Товарищи, мне пришла в голову замечательная идея: давайте составим список с объяснениями математических манипуляций толстых и зеленых, дабы на бордах торжествовала математическая справедливость.
>> No.57760 Reply
>>56911

бамп
>> No.57774 Reply
>>57734
Да, отличная идея.
>> No.57776 Reply
File: Troll_mathemathics_pi.jpg
Jpg, 33.71 KB, 350×494 - Click the image to expand
edit Find source with google Find source with iqdb
Troll_mathemathics_pi.jpg
>>57734
1) 0.(9) != 1
2) 0 * 1 = 0 * 2 => 1 = 2
3) пикилейтед
>> No.57798 Reply
Можно ли имея данные (около 100 значений) по 4 величинам получить функцию, связывающую одну из этих них с тремя другими так: x=a^n1b^n2c^n2?
>> No.57805 Reply
>>57734
16-36=25-45
16-36+81/4=25-45+81/4
(4-9/2)^2=(5-9/2)^2
4-9/2=5-9/2
4=5 => 2+2=5
>> No.57810 Reply
>>57805
> 16-36=25-45
> 16-36+81/4=25-45+81/4
Поясните пожалуйста, в чём суть?
>> No.57811 Reply
>>57810
> (4-9/2)^2=(5-9/2)^2
(8/2-9/2)^2 = (10/2 - 9/2)^2
±1/2 = ±1/2, а вовсе не
> 4-9/2=5-9/2
>> No.57822 Reply
>>57776
1) Необходимы точные понятия о термах 0,(9) и 1. В стандартной аксиоматике для R о 0,(9) речи не идёт вовсе.
2) В теореме о сокращении говорится о сокращении на ненулевой множитель или ненулевое множимое.
3) Не скажу, что знаток в этом вопросе. Со школы знаю, что за длину дуги окружности принимается предел последовательности длин описаных ломаных. Звенья описаных ломаных являются отрезками касательных, но в даном случае таких звеньев только четыре - слева, сверху, справа и снизу.
>> No.57825 Reply
>>57776
3) По ходу, у ломаной на рисунке множество точек излома несчетно и (возможно) имеет ненулевую меру. Она и добавляется к \pi, и получается большее число.
>> No.57826 Reply
File: bbi.ru.pdf
Pdf, 2.29 KB, 612×792 - Click the image to get file
bbi.ru.pdf
>>57776
> 1) 0.(9) != 1
Для этого нужно построить осмысленную биекцию между множеством десятичных записей, не оканчивающихся на 9999..., и множеством действительных чисел. Потом поговорить о суммах рядов с членами 9/10^k и осуществить отождествление на основании этого.
> 2) 0 * 1 = 0 * 2 => 1 = 2
Это совсем уж элементарно. Простейшее объяснение: для того, чтобы убрать лишний повторяющийся в обоих частях равенства сомножитель нужно домножить равенство на такое число, которое с этим сомножителем даст 1. Такое число называется обратным. У нуля нет обратных, а всякое число, умноженное на нуль, - нуль.
В общем, нужно школьные рецепты выразить на языке более менее нормальной алгебры.
> 3) пикилейтед
Это уже более серьёзный вопрос. Прикладываю книжку. Собственно, это в ней есть. Нужно ещё добавить интуитивное пояснение, почему так происходит. Примеры из Олмстеда-Гелбаума (глава 10) неспрямляемых кривых.

Также нужно убрать глупости типа "0/0 - неопределенность [всякая чушь про пределы]", т.е. ввести понятие корректности (осмысленности). Вместе с этим необходимо подавать понятие о мотивировке определения (собственно, зачем мы утверждаем какое-то определение),что входит в элементарную математическую культуру.
>>57822
> предел последовательности длин описаных ломаных.
Вписанных.

>>57734-кун
>> No.57831 Reply
>>57826
> Для этого нужно построить осмысленную биекцию между множеством десятичных записей, не оканчивающихся на 9999..., и множеством действительных чисел. Потом поговорить о суммах рядов с членами 9/10^k и осуществить отождествление на основании этого.
Но зачем? Ведь можно просто показать противоречие с аксиомой полноты на множестве действительных чисел.
>> No.57833 Reply
File: Задача2.JPG
Jpg, 12.16 KB, 470×300 - Click the image to expand
edit Find source with google Find source with iqdb
Задача2.JPG
Вот я в интернете нашел примеры для 2х2 матрицу на тему того, как найти собственное значение оператора, вот только когда 3х3 как быть, берем диагональ и отнимаем другие показатели или что
>> No.57836 Reply
>>57833
a-λ * *
* b-λ *
* * c-λ
Вычисляешь определитель, получаешь характеристический многочлен, находишь его корни.
>> No.57838 Reply
>>57776
Дуга не равно прямая линия.
Сколько бы мы не добавляли новых углов, но совпадать с линией окружности фигура с бесконечным количеством углов не будет. Будет оставаться маленький зазор.
Гуманитарий-кун.
>> No.57839 Reply
>>57836
В начале нахожу определитель, а потом делаю так?
a-λb-λa-λ - |A| = 0
>> No.57840 Reply
>>57839
ой
с-λ т.е. последнее
>> No.57841 Reply
>>57839
> В начале нахожу определитель, а потом делаю так?
Нет. Вычисляешь определитель по правилу, например, треугольников (YouTube: Определитель матрицы - Правило треугольника ), получаешь кубическое уравнение.


На картинке, которую ты приложил, определитель выглядит как произведение элементов на главной диагонали минус произведение элементов на побочной диагонали:
|A| = (3-λ)(3-λ) - 4*5 = (3-λ)^2 - 20.
Корни этого многочлена 3±2√5. Они и будут собственными числами.
>> No.57842 Reply
>>57838
> Сколько бы мы не добавляли новых углов, но совпадать с линией окружности фигура с бесконечным количеством углов не будет.
Так мы же к пределу перешли, не?
>> No.57843 Reply
>>57826
> Вписанных.
Ебать, Будто разница есть какая-то.
>> No.57857 Reply
>>57842
Не. Если кривая на каждом шаге кусочно-гладкая, то, переходя к пределу, гладкую кривую мы не получим никогда. У нас же в пределе получается кривая с нарушением гладкости (изломом) в каждой точке.
В общем, не стоит верить глазам. Если одна фигура очень похожа на другую, то это не значит, что они совпадают.
>> No.57860 Reply
>>57857
> Если кривая на каждом шаге кусочно-гладкая, то, переходя к пределу, гладкую кривую мы не получим никогда.
> В общем, не стоит верить глазам.
Прежде всего, дорогой друг, не стоит верить интуиции.

В данному случае мы имеем не просто сходимость, а равномерную сходимость, которая более богата результатами. Тем не менее, она может забывать "плохие" свойства. Давайте вспомним пример непрерывной, но нигде не дифференцируемой функции. Как она задавалась? Суммой ряда, сходившегося равномерно, зубчатых функции всё меньших и меньших размеров. Если бы последовательность членов ряда не сходилась к функции-константе 0, то ни о какой непрерывности бы и речи не могло быть - ряд бы просто расходился.
>>57831
> Но зачем? Ведь можно просто показать противоречие с аксиомой полноты на множестве действительных чисел.
Не определяя смысла десятичной записи, доказывать что-либо нельзя. А вот аксиома полноты как раз и будет работать, когда показываем, что ряд \sum_{k=1}^\infty 9/10^k сходится к единице.
>> No.57867 Reply
>>57860
> В данному случае мы имеем не просто сходимость, а равномерную сходимость, которая более богата результатами. Тем не менее, она может забывать "плохие" свойства. Давайте вспомним пример непрерывной, но нигде не дифференцируемой функции.
Но ведь в твоём случае как исходные функции у нас были не гладкими, так и результирующая оказалась не гладкой?

Под своими словами я имел в виду, что кусочно-гладкие функции, конечно, сходятся, но они сходятся не к той самой окружности, а к "плохой" кривой. А длина кривой может считаться только при условии непрерывной дифференцируемости.
>> No.57868 Reply
>>57867
> так и результирующая оказалась не гладкой?
Нуль не гладкий? По-моему, очень даже гладкий, аналитический, я бы сказал. (Речь, правда, не сумме ряда, а о пределе последовательности членов этого ряда.)
> они сходятся не к той самой окружности, а к "плохой" кривой.
К чему хотят к тому и сходятся. Теорема о том, что негладкие функции сходятся к негладким, неверна. Всюду разрывные функции могут сходиться к всюду непрерывным даже.
>> No.57885 Reply
>>57776
> пикилейтед
В чём фокус?
>> No.57887 Reply
>>57868
Ладно, не буду спорить. Захожу на минутку, сходу могу чушь ляпнуть... оу, и правда, вопрос же не в сходимости, а в ДОСТИЖИМОСТИ предела. Моя вина.

Пойдём более благоразумным путём. То, что предел данной ломаной не является длиной, доказывается тем, что по определнию длина - это предел той ломаной, чьи точки излома лежат на кривой. У нас же имеется бесконечно много точек, не лежащих на кривой (вершины треугольничков). Следовательно, длина данной на рисунке ломаной не будет сходиться в длину кривой по определению, а к ней будет сходиться предел "оснований" треугольничков.

Простите за мой сленг, я идиот.
>> No.57892 Reply
>>57885
В том, что мы не может сделать круговую линию. Хоть как ни делай линию более круглой, сумма все равно не получится такой. В этом и весь фокус.
>> No.57901 Reply
>>57887
> Следовательно, длина данной на рисунке ломаной не будет сходиться в длину кривой по определению, а к ней будет сходиться предел "оснований" треугольничков.
Ну опять что-то не то. Теоремы о том, что только длины вписанных вписанных ломаных, сходящихся поточечно к кривой, сходятся к длине кривой, нет. Есть следующая теорема: если последовательность кривых поточечно сходится к данной кривой, то нижний предел последовательности длин не меньше длины кривой. Это мы и наблюдаем. А вот равен он или нет - в каждом случае своё.
> Простите за мой сленг,
Кстати, в какой области гуманитарий?
>> No.58080 Reply
>>57901
> Кстати, в какой области гуманитарий?
Бля, Если математика наука, то гуманитарная. А сленг у него противный, это да. Алсо, в посте >>57822 уже показано, что размышления велись в обход определения.
>> No.58088 Reply
>>58080
> Если математика наука, то гуманитарная
Считаю математику метанаукой, ибо её область исследования - точные методы мышления познавающего. Всего метанауки две: точная и неточна (философия). А гуманитарные науки относятся к прикладным, по-моему.
>> No.58138 Reply
File: Черняев А.Ф. - Основы русской геометрии - 2004.pdf
Pdf, 4.45 KB, 420×595 - Click the image to get file
Черняев А.Ф. - Основы русской геометрии - 2004.pdf
Маткуны, будьте няшами, сделайте пользу кафедре. Нужно же знать откуда ноги растут.

Реквест на годные книги по теме. Всевозможная гносеология и онтология.

Столкнулся с книгой Черняева А.Ф. "Основы русской геометрии", где в первой и второй главе автор качественно троллит диалектикой текущее (по его словам) положение дел в математике.
Что характерно троллит весьма конструктивно.

ФГМ сходу не просматривается (может он и у меня уже), нужна сторонняя критика.
>> No.58154 Reply
>>58138
> автор качественно троллит диалектикой текущее (по его словам) положение дел в математике.
Прочитал первую главу, смысла за горой белиберды не увидел. В чём троллинг?
>> No.58210 Reply
Добра всем итт.
Ни у кого не завалялось методички по линалу?
1 курс, курсач - решить 40 примеров. wolframalpha не катит, ибо нужно с решением, тобишь самому делать. Такие дела.
>> No.58218 Reply
>>58210
Сколько ВУЗов - столько линалов. Скажи, какие у тебя темы, или лучше покажи примеры.
>> No.58224 Reply
File: arifmetika_nachala.pdf
Pdf, 10.60 KB, 595×842 - Click the image to get file
arifmetika_nachala.pdf
Я просто оставлю это здесь.
>> No.58225 Reply
>>58224
Хотя наверное уже боян.
>> No.58239 Reply
>>58224
Шоита ещё за репка? Кто такой этот Говоров?
>> No.58248 Reply
File: 43723895.jpg
Jpg, 50.86 KB, 410×264 - Click the image to expand
edit Find source with google Find source with iqdb
43723895.jpg
>>58224
Знает кто-нибудь, всерьёз это или в шутку?
Алсо, эта книга попала не на ту кафедру.
>> No.58249 Reply
>>58239
Очевидно, крутой мужик.
>> No.58252 Reply
>>58224
Похоже на новый уровень толстоты, с другой стороны даже самый толстый толстяк не стал бы столько писать.
>> No.58253 Reply
>>58224
> Какъ i Откуда растутъ Чiсла?
> Пифагоръ въ гостяхъ у Деда Мороза и Снегурочки
I lol'd
>> No.58256 Reply
>>58252
Автор книги - кореш Чудинова.
>> No.58257 Reply
>>58218
Матрицы (Крамер, Гаусс, ФСР), комплексные числа (уравнения, модуль), подпространства (базис, размерность, определитель).
В двух словах.
>> No.58272 Reply
>>58252
Ты знаком с трудами Пифагора? Это толстота того же самого уровня. То есть и не толстота вовсе, а гностическое (в случае Говорова христианское) мировоззрение, взявшее на вооружение математику. Собственно когда гностики взялись за химию, получилась алхимия. Только вот математика во времена еще Пифагора была развита намного сильнее, чем химия даже в последующие века.
>> No.58273 Reply
>>58272
Но ведь математика Пифагора и математика тех же Бурбаков - две разные математики.
>> No.58363 Reply
>>58273
Да, конечно.
Полагаю, у нас достаточно квалифицированные кадры, чтобы в треде "Кафедра математики" не опускаться и до уровня, который позволителен в средних университетах (не Гарвард, Принстон, Стэнфорд, ...), без нужды помочь в случае реквеста, предполагающего именно такой взгляд.
>> No.58408 Reply
>>58257
http://vmtusur.narod.ru/book.html
Вот, посмотри тут.
>> No.58461 Reply
File: 0a1e06d5e416b0ebd1ebac0c8be0b8bf.jpg
Jpg, 668.47 KB, 2043×2885 - Click the image to expand
edit Find source with google Find source with iqdb
0a1e06d5e416b0ebd1ebac0c8be0b8bf.jpg
http://rghost.ru/37838511
Пожалуйста, добрый анонимус, помоги мне закончить. Я не понимаю, что делать дальше. Да, я тупой, там написана формула, но я все равно не могу.
Если тебе несложно.
Расплачусь тоннами Хоро.
>> No.58526 Reply
Математач, поясни по-хардкору.
Задание - решить слу по правилу Крамера.

2x1 + 2x2 + 11x3 + 5x4 = 2
x1 + x2 + 5x3 + 2x4 = 1
2x1 + x2 + 3x3 + 2x4 = -3
x1 + x2 + 3x3 + 4x4 = 6

Нашёл определители всех 5 матриц, которые долны получится (8, -226, -67, 0, -27). В результате
x1 = -28,25
x2 = -8,375
x3 = 0
x4 = -3,375
Что, собственно, является неправильным решением.
>> No.58530 Reply
>>58526
A =
{
{2, 2, 11, 5},
{1, 1, 5, 2},
{2, 1, 3, 2},
{1, 1, 3, 4}
}
det A = -4

A1 =
{
{2, 2, 11, 5},
{1, 1, 5, 2},
{-3, 1, 3, 2},
{6, 1, 3, 4}
}
det A1 = 26

A2 =
{
{2, 2, 11, 5},
{1, 1, 5, 2},
{2, -3, 3, 2},
{1, 6, 3, 4}
}
det A2 = -45

A3 =
{
{2, 2, 2, 5},
{1, 1, 1, 2},
{2, 1, -3, 2},
{1, 1, 6, 4}
}
det A3 = 5

A4 =
{
{2, 2, 11, 2},
{1, 1, 5, 1},
{2, 1, 3, -3},
{1, 1, 3, 6}
}
det A4 = -5

x1 = det A1/det A = -26/4
x2 = det A2/det A = 45/4
x3 = det A3/det A = -5/4
x4 = det A4/det A = 5/4
>> No.58533 Reply
File: 1.png
Png, 1.35 KB, 185×101 - Click the image to expand
edit Find source with google Find source with iqdb
1.png
>>58526
Пересчитай определители.
>> No.58535 Reply
>>58530
Я максимум зафейлил.
Спасибо, но каким методом решал?
Я - через разложение первой строки, ну и нахождение определителей матриц 3х3.
>> No.58536 Reply
>>58533
Каким методом?
Алсоу - программно посчитать не катит - нужно самому расписать.
Кроме вышеуказанного способа - не знаю ничего.
>> No.58538 Reply
>> No.58539 Reply
>>58536
Метод Гаусса используй.
>> No.58540 Reply
>>58536
Приведите вы гауссовским методом матрицу к диагональной, делов то.
>> No.58542 Reply
>>58539
>>58540
А потом? Ну, тоесть как при этом Крамера использовать?
>> No.58543 Reply
>>58542
Для использования метода Крамера для решения твоего примерчика тебе нужно знать пять определителей.
Метод Гаусса позволяет быстро находить определители: приводишь матрицу к треугольному виду с помощью элементарных преобразований, определитель будет равен произведению элементов, стоящих на главной диагонали.
Определители пяти матриц находишь по Гауссу, решение находишь по Крамеру.
>> No.58544 Reply
>>58542
Прошу прощения - что-то я люто торможу. Всё понял.
Только вот хотелось бы ручками определитель найти.
Если будет матрица ступенчатого вида - её определитель равен коэфициентам по диагонале?
>> No.58546 Reply
>>58543
> матрицу к треугольному виду с помощью элементарных преобразований, определитель будет равен произведению элементов, стоящих на главной диагонали
Вот, спасибо.
Что и хотел узнать.
Дефолтным методом разложения первой строки выходит "сегодня ты считаешь определитель, а завтра ты считаешь определитель".
>> No.58547 Reply
>>58536
Любым, хоть через перестановки. Через разложение первой строки: det [[2 2 11 5], [1 1 5 2], [2 1 3 2], [1 1 3 4]] = 2 * det [[1 5 2], [1 3 2], [1 3 4]] - 2 * det[[1 5 2], [2 3 2], [1 3 4]] + 11 * det[[1 1 2], [2 1 2], [1 1 4]] - 5 * det[[1 1 5], [2 1 3], [1 1 3]] = 2 * (-4) - 2 * (-18) + 11 * (-2) - 5 * 2 = -8 + 36 - 22 - 10 = -4
>> No.58548 Reply
Раз тут есть живые люди - понадоедаю еще.
ФСР СЛАУ должна из себя представлять (n-r) векторов какого вида?
Допустим, в результате преобразований, у меня получилось 2 уравнения:
2x1 + 5x2 + 3x3 - x4 = 0
x2 + 5x3 - 3x4 = 0

Следуя педивикии у меня должно выйти:
z1 = (-11, 3, 1, 0); z2 = (7, -5, 0, 1)
>> No.58551 Reply
>>58548
Начнем с того, что каждый вектор из ФСР сам по себе должен являться решением системы. У тебя это не так. Распиши, как решал.
>> No.58553 Reply
>>58551
СЛОУ:
x1 + 2x2 - x3 + x4
2x2 + 5x2 + 3x3 - x4
x1 + x2 - 6x3 + 4x4
x1 + 3x2 + 4x3 - 2x4

I+(-II); II+(-III); III*2+(-IV); I+(-II); II+III;
>> No.58555 Reply
>>58553
Я не про нее. А про то, как ты по
> 2x1 + 5x2 + 3x3 - x4 = 0
> x2 + 5x3 - 3x4 = 0
получил вектора z1,z3.
>> No.58556 Reply
>>58555
Выразил x1, x2 через x3, x4.
Потом какие-то антинаучные операции провёл с x3 и x4 - подставлял единицу и 0.
Пропустил оче много пар, ориентируюсь по википедии.
>> No.58558 Reply
>>58556
   x2 + 5x3 - 3x4 = 0
2x1 + 5x2 + 3x3 - x4 = 0
   x2 = -5x3 + 3x4
2x1 + 5x2 = -3x3 + x4


1) Подставляем x3 = 0, x4 = 1
x2 = 3
2x1 + 5x2 = 1

x2 = 3
x1 = -7
Первый вектор ФСР = (-7, 3, 0, 1)
2) Подставляем x3 = 1, x4 = 0
x2 = -5
2x1 + 5x2 = -3

x2 = -5
x1 = 11
Второй вектор ФСР = (11, -5, 1, 0)


Вроде нигде в расчетах не ошибся. Надеюсь идею ты понял.
>> No.58561 Reply
>>58558
Понял. Я просто в спешке, походу, не правильно просчитал. Только не понятная идея подставления 1 и 0. Ну да ладно, спасибо.
>> No.58569 Reply
>>58561
> идея подставления 1 и 0
Фундаментальная система решений - это, по википедии, набор линейно независимых решений однородной системы уравнений.
Пусть в некой системе есть две зависимые переменные x1, x2 выражаются через три свободные x3, x4, x5. То есть подставляя в x3, x4 и x5 любые вещественные числа, ты каждый раз будешь получать вектор (x1,x2,x3,x4,x5), являющийся будет решением этой системы.
Тебе нужно найти хотя бы один набор из трех таких векторов, которые будут линейно независимы между собой. Их число равно числу свободных переменных(n-r, как ты сам написал).
Сделать это очень просто. Просто берешь вектора вида
(x1, x2, 1, 0, 0)
(x1, x2, 0, 1, 0)
(x1, x2, 0, 0, 1) и вычисляешь в них зависимые переменные. Ранг такой матрицы равен рангу единичной матрицы в правой ее части, то есть трем. Следовательно все вектора линейно независимы(хотя это и так видно, ящитаю).
>> No.58588 Reply
File: 768769867987.JPG
Jpg, 15.54 KB, 922×194 - Click the image to expand
edit Find source with google Find source with iqdb
768769867987.JPG
Ответ верный, это точно, но как получился этот разрыв? Строю графики, все равно не доходит. Анон, помоги.
>> No.58590 Reply
File: 7adcb7986e73.jpg
Jpg, 65.28 KB, 480×458 - Click the image to expand
edit Find source with google Find source with iqdb
7adcb7986e73.jpg
Есть похожее задание, но все-таки понятнее не становится. Множеством решений все равно является отрезок.
>> No.58591 Reply
Может быть, должен получится непрерывный отрезок?
>> No.58618 Reply
File: x_c21bd860.jpg
Jpg, 14.76 KB, 240×320 - Click the image to expand
edit Find source with google Find source with iqdb
x_c21bd860.jpg
Добрый вечер! У меня просьба , реши пожалуйста вот это:
Нати длину вектора С=А-2В, если |А|=2, |В|=3, АˆВ=60

И если можно разясни поподробнее , а то самому сложно понять, так как изучаю по учебникам , заранее спасибо
>> No.58619 Reply
>>58618
> АˆВ
Чтоэта?
>> No.58621 Reply
>>58619
Угол
>> No.58623 Reply
>>58619
Угол между векторами.
Короче, у тебя получается треугольник со сторонами А, 2В и С.
Можешь посчитать по теореме Косинусов сторону С: |С|^2 = |A|^2 + |B|^2 - 2*|A||B|cos(A^B)
>> No.58640 Reply
File: y_25605fc6.jpg
Jpg, 109.26 KB, 602×807 - Click the image to expand
edit Find source with google Find source with iqdb
y_25605fc6.jpg
>>58623 У меня видимо пробелы в теории , но скажи пожалуйста как соотносятся |А|(модуль) и просто А(вектор), ведь даны только модули? Из формулы кос я могу найти АВ(произведение векторов) ,а не чему они равны в отдельности?
>> No.58643 Reply
>>58461
Ребята, ну халп же!
>> No.58644 Reply
>>58643
Ты сам-то знаешь вообще, что хочешь? Я, например, нихуя не понел из твоей таблицы кроме того, что ты обрабатываешь какую-то статистику. Что такое бета-коэф. я не в курсе.

>>58640
Что ты несёшь вообще? Модуль вектора есть его длина (без направления), из формулы косинусов ты получишь прямой ответ на твой вопрос (если заменишь В на 2В, например). Чему равны вектора в отдельности ты никогда не узнаешь, потому как вся система этих векторов инвариантна относительно вращений и перемещений в заданной с.к.
Или ты не понел, что тебе нужно
> Нати длину вектора С
а не его самого?
>> No.58646 Reply
>>58644
Там внизу прописаны формулы, нужно произвести расчеты, пользуясь значениями, приведенными в таблице.
Да, я гуманитарий, кстати.
Это ад пизды для меня.
>> No.58649 Reply
>>58646
> я гуманитарий
Это оправдание идиотизма? Кстати, уже проведённые расчёты сделаны не тобой ведь, да?
>> No.58650 Reply
>>58649
Ну да, не мной, совершенно верно. Если тебе не очень сложно помочь, помоги, очень тебя прошу. Мне осталось что-то около часа до сдачи.
>> No.58651 Reply
File: 3f216ad235155be3f0e4034e5f767142.png
Png, 90.14 KB, 600×600 - Click the image to expand
edit Find source with google Find source with iqdb
3f216ad235155be3f0e4034e5f767142.png
>>58650
Я тебе помогу. У тебя есть значения в таблицах. Ты берёшь эксель (если не умеешь в нём писать формулы - то калькулятор) и по написанным там же формулам вычисляешь всё что нужно.
В графе
> Average
находятся величины с чертой сверху (это символ усреднения).
Кстати, формулы для сигм написаны неверно, как я понимаю. Там должно быть суммирование по отклонениям для каждого измерения.
Удачи.
>> No.58652 Reply
>>58650
Черт побери, доброматематики, помогите, пожалуйста. До сдачи осталось всего-ничего, в благодарность могу помочь всем, чем могу.
>> No.58653 Reply
>>58651
Спасибо за пожелания, добряк. вообще, конечно, ты оказал бы мне услугу еще большую, если бы посчитал это нехитрое дело и выложил на рыгост, ибо у меня с этим действительно большие проблемы, а до сдачи меньше получаса
>> No.58661 Reply
Анон, а можно ли одиннадцатикласснику к экзамену подучить математику по учебнику Фадеева? А то был недавно на лекции Востокова про теорему Ферма и историю её решения, понял одну десятую часть и задумался что мало что понял. (Хотя лекция и была явно повыше уровнем, чем просто для выпускника средней школы)
>> No.58696 Reply
>>58661
Что за книга? Дай посмотреть, а то непонятно о чём речь. Что-то навроде "Подготовка по C%number% ЕГЭ", полагаю?
>> No.58773 Reply
File: x_ee91faea.jpg
Jpg, 40.33 KB, 500×375 - Click the image to expand
edit Find source with google Find source with iqdb
x_ee91faea.jpg
>>58644> Спасибо большое , разобрался с векторами
>> No.58777 Reply
>>58696
Университетский учебник с матмеха СПБГу
Капча говорит "нашему прознали" - не знаю к чему.
>> No.58783 Reply
>>58777
Можно конечно, но не нужно. На самом деле Фадеев не очень хорошая (скорее даже плохая) книга, на матмехе по ней занимаются только непонятные специальности типа "прикладная математика в МО". По АТЧ лучше почитать Винберга, например.
>> No.58870 Reply
найти первые частные производные функции в точке 0,0 - значит взять производные, а затем подставить 0 вместо х и у? Тогда в чем смысл, получается 0 в f'x и f'y
>> No.58875 Reply
>>58870
> Тогда в чем смысл, получается 0 в f'x и f'y
Иной раз получается, а иной раз нет. Функции, они ведь разные.
>> No.58917 Reply
Добрый вечер, есть задания с функц. неск. переменных.
Надо вычислить приближенно с помощью дифференциала. Как делать с одной переменной я знаю, а как быть с несколькими, вот допустим есть функция. cos^3(3.02^2 - 2.99^2), Т.е. мне вычислить по отдельности каждую и затем отнять или как?
>> No.59056 Reply
Сап, матемач, требуется помощь в решении интеграла численно.

http://articles.adsabs.harvard.edu//full/1983MNRAS.203..735C/0000737.000.html (формула 4).
Что мы имеем в данный момент:
Вместо вот этой штуки (http://puu.sh/t6Ms) мы подставили http://www.wolframalpha.com/input/?i=D[Q0*exp%28-r%2Fh%29*%28cosh[z%2Fz0]%29^%28-2%29%2Cr]

Далее, преобразовали вот так:
http://puu.sh/t4Z9

В конечном счёте получилось вот это: http://puu.sh/t6Za

Появилось несколько вопросов.
1. Можно ли было вносить переменную интегрирования u внутрь второго интеграла? (http://puu.sh/t6XW)
2. Можно ли заменять пределы интегрирования [0;\infty] на [0;tan(pi/2)] (больше изменений никаких не было внесено, замены переменных не было, только пределов)?
3. Реально ли посчитать данный интеграл аналитически?

Интеграл решаем с помощью метода Монте-Карло. Вот код: http://pastebin.com/QKsH1Mkb (часть на C#, часть на чистом си -- можно не обращать внимание).
>> No.59206 Reply
File: 1334154075974.jpg
Jpg, 960.11 KB, 1920×1080
edit Find source with google Find source with iqdb
1334154075974.jpg
File: 27a2e33174327966b...
Jpg, 245.90 KB, 748×1072
edit Find source with google Find source with iqdb
27a2e33174327966bf50b6738d3f689d.jpg
File: 2c959322d382deca2...
Jpg, 166.40 KB, 1000×1000
edit Find source with google Find source with iqdb
2c959322d382deca2ceca74bd849e267.jpg

>>59056
Типа бамп.
>> No.59305 Reply
Аноны, сколько времени у вас уйдёт на эту простенькую систему уравнений?
x+y=1
z+y=2
f+x=3
z+p=4
f+n=0
n+k=p
>> No.59334 Reply
Доброанон, у меня к тебе две (точнее полторы) просьбы. Расскажи сам или кинь годных книжек по численным методам. Сейчас меня интересует построение сеток для двумерных краевых задач. С сетками на прямоугольных областях я разобрался, а еще мне надо научится делать это же для области, у которой отрезан один угол. Там начинается какая-то фигня, по лекциям никак не разберусь.
>> No.59338 Reply
>>59334
А в чём проблема? Нельзя просто ломаной из клеточек приблизить этот кусочек границы?
>> No.59340 Reply
>>58783
А Мордкович в общем хороший автор?
>> No.59341 Reply
File: 2e039440e74b70d3dfb5901fdce40ec13200c594.jpg
Jpg, 267.20 KB, 700×700 - Click the image to expand
edit Find source with google Find source with iqdb
2e039440e74b70d3dfb5901fdce40ec13200c594.jpg
>>59340
Нет, когда его читал было желание взять и закрыть учебник, когда готовился к ЕГЭ брал информацию в интернете, да и вообще теорию Мордковича я открывал пару раз в жизни, не понравилось.
>> No.59350 Reply
Посоны, извиняйте за школьный вопрос, но я совершенно не знаю, как вынести корень из-под корня. Например sqrt(2 + sqrt(3)). Расскажите алгоритм штоле. Полный квадрат искать?
>> No.59351 Reply
File: Безымянный.png
Png, 27.09 KB, 1351×549 - Click the image to expand
edit Find source with google Find source with iqdb
Безымянный.png
>>59350
Два раза возвести в квадрат?
>> No.59352 Reply
>>59340
Слабоват, вроде. Я выбираю Виленкина, Ивашева-Мусатова, Шварцбурда.
>> No.59354 Reply
>>59305
Числа целые? Отрицательные + положительные + 0 ?
>> No.59391 Reply
File: 0f649c7891ab93076...
Jpg, 194.72 KB, 602×850
edit Find source with google Find source with iqdb
0f649c7891ab93076...
File: 7a9937d48fea3522f...
Jpg, 148.93 KB, 620×715
edit Find source with google Find source with iqdb
7a9937d48fea3522f7d55fb42fdcbcf3.jpg
File: 1d22b56909e498b10...
Jpeg, 220.71 KB, 600×800
edit Find source with google Find source with iqdb
1d22b56909e498b10...

>>59305
Количество переменных > количество уравнений. Решений может быть бесконечно много. Если же какую-нибудь переменную определить, то систему можно решить методом Гаусса или даже вручную, просто выражая переменные одну через другую:

p = 0

x = 1 - y
x = 3 - f
1 - y = 3 - f ==> y = f - 2
y = 2 - z
2 - z = f - 2 ==> f = 4 - z
f = 3 - x
3 - x = 4 - z ==> z = x + 1
z + 0 = 4 ==> z = 4

z = 4
y = -2
x = 3
f = 0
n = 0
k = 0

-----------
~ 6-8 минут.

Алсо, бамп >>59056
>> No.59413 Reply
File: Безымянный.PNG
Png, 16.47 KB, 1008×630 - Click the image to expand
edit Find source with google Find source with iqdb
Безымянный.PNG
Теперь мне понятен уровень ЭКСПЕРТОВ в этом треде. Иди алгебру учи, лолка.

Ах да, и по теме треда - если кому нужен учебник, выбирайте Виленкина. Этот питушок хоть и сложно пишет, но понять можно.
>> No.59414 Reply
Ах да, это >>59413
ему - >>59351
>> No.59422 Reply
>>59391
Ну, как видишь, целые. Правда ты к чему это? Решение одно.
>>59391
Не-а. Оно решается без определения какой-нибудь переменной.
>> No.59428 Reply
File: 2ed25864c29aec20e1ab8ecf5d344a48.jpg
Jpg, 773.76 KB, 1600×1142 - Click the image to expand
edit Find source with google Find source with iqdb
2ed25864c29aec20e1ab8ecf5d344a48.jpg
>>59422
Объясните мне, пожалуйста, как можно решить систему уравнений, в которой переменных больше, чем самих уравнений, да ещё в единственном решении?
К примеру:
x + y + 1 = 0
y + z = 0
и чему же здесь будет равно x, y, z? Строго говоря, решений здесь бесконечно много (например, x,y,z = 0, -1, 1; 1, -2, 2; 2, -3, 3; ... ). Единственное, о чём я могу подумать, это "зафиксировать" какую-нибудь переменную, а относительно неё уже можно решать.
Например, y = const.
Тогда,
x = -y - 1
z = -y
Целые числа в решении получились от того, что ни в одном уравнений нет операций умножения, деления, возведения в степень, к тому же я выбрал в качестве переменной p целое число (а мог взять дробное или даже комплексное).
>> No.59433 Reply
>>59422
Уравнений 6, а переменных 7 - одна - линейная комбинация остальных, либо все выражаются через одну.
Дальше вольфрам:
http://www.wolframalpha.com/input/?i=x%2By%3D1+z%2By%3D2+f%2Bx%3D3+z%2[...]k%3Dp
>> No.59591 Reply
File: 0a618e4dbd02379f3...
Jpg, 413.46 KB, 715×1000
edit Find source with google Find source with iqdb
0a618e4dbd02379f3afb1b999d4ec68b.jpg
File: 429ef794c2ab8b687...
Jpg, 953.96 KB, 792×1120
edit Find source with google Find source with iqdb
429ef794c2ab8b687d615c09647d6f6d.jpg
File: 1336583312228.jpg
Jpg, 86.62 KB, 1280×720
edit Find source with google Find source with iqdb
1336583312228.jpg

>>59433
Я немного подзабыл аналитическую геометрию (или линейную алгебру). Напомните мне, как можно проверить систему, имеет ли она линейно-зависимые строки в матрице?

Из системы уравнений это не очевидно, а что, если система является линейно-независимой?
Алсо, что-то я не понял инпута в вольфраме. Логичнее был вот такой вот инпут: http://www.wolframalpha.com/input/?i=solve+x%2By%3D1%3B+z%2By%3D2%3B+f[...]k%3Dp

Но здесь он, как я и сказал выше, решил систему относительно p.
>> No.59592 Reply
>>59591
ну окей. Если я лин.ал. не забыл, то это теорема Кронекера-Капелли
>> No.59700 Reply
Является ли функция дифф-й в точке (0.0)
Т.е. мне надо в начале взять частные производные по х и по у, а затем подставить 0?
>> No.59844 Reply
File: 62ba844a725de3938c01b9543b3768f0.jpg
Jpg, 120.82 KB, 559×593 - Click the image to expand
edit Find source with google Find source with iqdb
62ba844a725de3938c01b9543b3768f0.jpg
Чему равен arctan 0,5?
inb4: 0,46364760900080611621425623146121
>> No.59845 Reply
>>59844
Вольфрам говорит, что это ~26.6 градусов
>> No.59847 Reply
>>59845
Мне бы тригонометрическое представление, через Pi.
Например: arctan 1 = Pi/4
>> No.59849 Reply
>>59847
Окей, не знаешь как перевести в радианы?
http://www.wolframalpha.com/input/?i=arctg%280.5%29+in+radians
>> No.59853 Reply
>>59849
Ты меня просто тролируешь.
Я предполагаю, что arctan 1/2 = Pi/8
>> No.59854 Reply
>>59853
Блин, глупыш ты. 180 градусов - Pi радиан.
Но радиан не говорят в таких единицах обычно.
>> No.59875 Reply
> arctan 1/2 = Pi/8
Не линейная же функция.
Алсо, нетабличное значение.
>> No.59911 Reply
>>59844
>>59845
>>59847
>>59849
>>59853
>>59854
Позабавили.
>>59889
http://eqworld.ipmnet.ru/ru/library/mathematics.htm
http://eek.diary.ru/p0.htm?oam#more13
Не знаю, насколько это хорошо, но что пришло в голову уж. Первый выводится как первый рез в гугле по запросу "книги математика"
>> No.59913 Reply
>>59889
Книжки по всему есть. По математике многие есть, даже 2012 году выпущенные.
http://libgen.info/
>> No.59914 Reply
>>59844
223/480, 11/24 если совсем грубо.
http://www.wolframalpha.com/input/?i=arctan+Taylor
>> No.59915 Reply
>>59914
Ему нужно точное значение.
>> No.59945 Reply
>>59911
>>59913
Спасибо, лучи добра вам.
>> No.59967 Reply
Анончик, вот смотри:
Мне нужно получить уравнение движения точки вокруг оси, например в 2D, то это можно представить так
x(t)=x0 + rcos(w t)
y(t)=y0 + rsin(w t)
или через линейный операторы, матрицу поворота:
P(t)=P0 + M(wt)r
Однако, в обоих случаях я по прежнему должен работать с тригонометрическими функциями sin, cos, а для моей задачи, решение с их использованием хрен найдёшь.

Т.е. мне нужен способ всё свести к линейной алгебре, вообще без этих проклятых функций. Собственно, о том и вопрос. Как это вообще можно сделать? И какие накладываются ограничения.

Мои потуги подсказывают мне ,двигаться в сторону векторного произведения т.е. получить вектор скорости v = [w,r], s = v + r, r' = (s / |s|) * |r| (я поучаю вектор r' уже повёрнутым, а само произведение получать из косометрической матрицы: [w,r] = [w] * r) Но беда в том что тут нету ни какой взаимосвязи со временем, что не торт.

Если есть идеи, подскажи анон, ЧЯДНТ?
>> No.59970 Reply
>>59967
> точки вокруг оси,
> например в 2D
Наркоман?
> в сторону векторного произведения
> в 2D
Ещё веселее.
> косометрической
Геоморфизм-кун?
>> No.59975 Reply
File: 1248390964009.jpg
Jpg, 16.89 KB, 360×362
Your censorship settings forbid this file.
unrated
>>59970
в 2D вырождается в точку.
> Ещё веселее.
Не тупи, 2D было только в рамках примера, основная задача:
> нужно получить уравнение движения точки вокруг оси
Кососимметричная, кососимметрическая и.т.д
ruwiki://Кососимметричная_матрица
>> No.59994 Reply
File: 6KmAm2M4h5M.jpg
Jpg, 354.03 KB, 1280×1024
edit Find source with google Find source with iqdb
6KmAm2M4h5M.jpg
File: 0KRvEnIF5OU.jpg
Jpg, 361.24 KB, 1280×1024
edit Find source with google Find source with iqdb
0KRvEnIF5OU.jpg
File: 68e4b1ffc8cd0d1a3...
Jpg, 1346.53 KB, 3296×2334
edit Find source with google Find source with iqdb
68e4b1ffc8cd0d1a3209c1814d8fb2fd.jpg

>>59056
Лютый, бешеный бамп.
>> No.60007 Reply
File: Снимок.PNG
Png, 15.44 KB, 605×245 - Click the image to expand
edit Find source with google Find source with iqdb
Снимок.PNG
Анон, взял скрипт из вики, но не пойму в чем ошибка. Объясни ньюфагу. Хочу сделать процесс ортогонализации Грама–Шмидта. Если есть другие варианты автоматизированно это сделать, буду рад увидеть.
>> No.60018 Reply
>>59056
1. Можно, это теорема Фубини.
2. Не понял нахера это делать.
3. p зависит как-то от u и z? Если да, то надо знать как.
Если нет, то вынеси его нафиг из интеграла и посчитай (только оно разойдется у тебя, потому что корень из u расходится).

Кроме того я не понял трюка с Q0. В статье там rho(r, z) как-то не так выражается через r и z. В смысле Q0 там какое-то разное бывает.
>> No.60023 Reply
File: 238951178adb8fafa...
Jpg, 68.94 KB, 440×870
edit Find source with google Find source with iqdb
238951178adb8fafa93ddc895b5ecaff.jpg
File: c1b1a485d0343c1dd...
Jpg, 100.81 KB, 530×650
edit Find source with google Find source with iqdb
c1b1a485d0343c1dde71f8d5599f9fcc.jpg
File: e0e793ea5b3e3c8ea...
Jpg, 732.20 KB, 1200×1200
edit Find source with google Find source with iqdb
e0e793ea5b3e3c8ea588e4e54e5b1870.jpg

>>60018
2. Чтобы избежать счёта до бесконечности на ЭВМ: в данном случае можно задать шаг наподобии tg(pi/2 * delta), где delta \in [0; 1]
3. В приведённой ссылке на странице 738 есть расшифровка p:
p = x - sqrt(x^2 - 1)
x = (r^2 + u^2 + z^2) / (2 * r * u)
r = const.
Q0 = const.
>> No.60246 Reply
Ради увеселения на кафедре начинается трансляция задач с одного известного места (http://www.math.harvard.edu/putnam/).
http://mathbin.net/97152
>> No.60458 Reply
File: dolan.png
Png, 30.29 KB, 318×437 - Click the image to expand
edit Find source with google Find source with iqdb
dolan.png
>>59967
Я не специалист, но мне кажется, что зависимость оператора от времени - нелинейная функция, как ты собрался сделать из композиции с ней линейную?
>> No.60473 Reply
File: 45361.jpg
Jpg, 244.05 KB, 1024×768 - Click the image to expand
edit Find source with google Find source with iqdb
45361.jpg
>>60458
> как ты собрался сделать
Уже и не собираюсь. Похоже это будет действительно не возможно. Из своей задумки я получил формулу поворота во круг оси в n-мерном пространстве, но и там тригонометрия будет. Могу даже примерно всё рассчитать без неё, но там получится рекуррентная формула, которая ни чем не лучше. Печаль в общем.
>> No.60502 Reply
File: 1320433288430.png
Png, 995.18 KB, 828×1056 - Click the image to expand
edit Find source with google Find source with iqdb
1320433288430.png
Как подготовиться к зачету по матану и линалу за 3 дня?
Умеет ли вольфрамальфа в дифференциальное исчисление функций нескольких переменных (в т. ч. дифференцирование неявных функций)? Если да, то как?
>> No.60514 Reply
>>60502
А какая проблема? Зачёт семестровый, предмет матан&линал 2 в 1 или каждый по отдельности?
В общем случае схема такова: стул, учебник, стол опционален.
Умеет: введи differentiate 1/2y=x^2-5x
Для частных производных, ну например: d/dx x^2 y^4, d/dy x^2 y^4
>> No.60522 Reply
File: scullysmiling.gif
Gif, 771.08 KB, 320×180 - Click the image to expand
edit Find source with google Find source with iqdb
scullysmiling.gif
>>56360
Анончик, это ведь конспект из НМУ? Никак не могу найти остальные части. Доставь моар, будь няшей
>> No.60560 Reply
File: ss-(2012-05-18-at...
Png, 13.49 KB, 641×232
edit Find source with google Find source with iqdb
ss-(2012-05-18-at-05.15.56).png
File: ss-(2012-05-18-at...
Png, 10.26 KB, 668×149
edit Find source with google Find source with iqdb
ss-(2012-05-18-at-05.16.38).png

>>60514
семестровой, 2ой семестр, по отдельности.

По поводу вольфрамальфа:
Что делать, если номер что-то типа пикрелейтеда?
>> No.60570 Reply
>>60560
Демидович, страница 312?
>> No.60577 Reply
>>60570
Демидович, страницу не помню.
>> No.60586 Reply
>>60560
В первом случае пишешь напрямую: differentiate уравнение.
Во втором, тут нужно differentiate и то же самое уравнение, только вместо x - x(z). В 3401: differentiate x(z)+y+z=0
И differentiate (x(z))^2+y^2+z^2=1

У вас за семестр были дифф. операторы, тройные интегралы, контурные интегралы, формула Стокса и т.д.?
>> No.60591 Reply
>>60522
http://ium.mccme.ru/f06/an.html
http://ium.mccme.ru/f06/an_ex.html
http://ium.mccme.ru/s07/an.html
http://ium.mccme.ru/s07/an_ex.html
http://ium.mccme.ru/f07/an3s.html
http://ium.mccme.ru/f07/an_ex.html
Из этого в точности состоит книжка С. М. Львовского. Та лекция чуть устарела (вроде бы 1999 года).
>> No.60609 Reply
File: 4pc_enl.JPG
Jpg, 294.41 KB, 800×600 - Click the image to expand
edit Find source with google Find source with iqdb
4pc_enl.JPG
Доброанон, прошу тебя посоветовать мне учебник по математике. Причём такой, в котором основной упор делался бы на объяснение предмета и донесение математических истин до понимания, представления, а не на длинные формальные выкладки. Если такое возможно.
Собираюсь "привести ум в порядок", но думаю начать с чего-нибудь удобоваримого.
inb4 учебник для первоклашек%
>> No.60622 Reply
>>60586
нет, не было
>> No.60625 Reply
>>60609
> учебник по математике
Извини, учебников по математике не существует, либо они бесполезны - Бурбаки. Мне встречались учебники по разным направлениям математики.
> Причём такой, в котором основной упор делался бы на объяснение предмета и донесение математических истин до понимани
Для этих целей, думаю, книги не подходят. Найди у себя в вузе какого-нибудь более или менее нормального преподавателя математики, не удовлетворенного собственной студенческой аудиторией. Возможно, он сможет тебя консультировать. Даже продвинутые изучающие математику могут неправильно понять написанное в книжке. Здесь никого нормального нет. Я не нормальный, если что. Ещё скооперируйся со своими однокурсниками твоего уровня, чтобы какую-то организованную математическую деятельность проводить.
Учебники? Хм... по твоему запросу подойдут эти но я, возможно, ошибаюсь насчет твоего объема знаний:
Гельфанд-Шень "Алгебра";
Зельдович-Яглом "Высшая математика для начинающих физиков и техников";
Прасолов "Задачи по алгебре, арифметике и анализу";
Алексеев "Теорема Абеля в задачах и решениях".
>> No.60653 Reply
File: MSP12271a1cff093h4c7g3200002ehc0befhb7e0a2d.gif
Gif, 0.75 KB, 130×24 - Click the image to expand
edit Find source with google Find source with iqdb
MSP12271a1cff093h4c7g3200002ehc0befhb7e0a2d.gif
>>55775
Суп, анон. Готовлюсь к экзаменам и никак не могу решить пикрилейтед. Требуется найти x при которых получиться геометрическая прогрессия.
>> No.60657 Reply
>>60653
Найди знаменатель прогрессии. Их тут 2. Потом приравняй.
>> No.60659 Reply
Аноны, никак не могу разобраться с методом решения СЛАУ с помощью метода отражений Хаусхолдера. Надо именно привести в верхне-треугольной форме, может кто подкинет примерчик? А если ещё есть реализация алгоритма то буду очень рад исходникам
>> No.60661 Reply
Кто-нибудь может привести пример недиагонализируемой матрицы? Т.е. такой квадратной матрицы A, для кот. не существует такой матрицы T: T^(-1)AT = диагональной м-це?
>> No.60662 Reply
>>60661
уже не надо, нашел такую
1 0
1 1
>> No.60663 Reply
>>60653
x=9
Всё просто - по смыслу каждый член должен отличаться от последующего на какой-то множитель. Строишь систему:
sqrt(x+7)*k=sqrt(x-5)
sqrt(x-5)*k=1

Выражаешь через k, решаешь уравнение, где sqrt(x+7)=x-5, и в итоге x=9.
Множитель равен 1/2.
>> No.60677 Reply
File: ня-anime-песочница-57947.jpeg
Jpeg, 36.27 KB, 604×340 - Click the image to expand
edit Find source with google Find source with iqdb
ня-anime-песочница-57947.jpeg
>>60657
>>60663
Спасибо аноны, уже понял.
>> No.60685 Reply
File: as.gif
Gif, 1.43 KB, 245×36 - Click the image to expand
edit Find source with google Find source with iqdb
as.gif
>>60677
И снова я аноны. Делаю упражнение с тем же требованием. Найти x при которых получиться геометрическая прогрессия.
Перевожу пикрилейтед1 в пикрилейтед2 и решаю его, получаю x = 10, что собственно правильно, но в ответах моей книжки есть 2 ответа 10 и 0.0001.
Где затаился второй ответ?
>> No.60686 Reply
File: asc.gif
Gif, 1.68 KB, 296×40 - Click the image to expand
edit Find source with google Find source with iqdb
asc.gif
Отклеился пикрилейтед_2
>> No.60691 Reply
File: 050_14.jpg
Jpg, 45.54 KB, 500×363 - Click the image to expand
edit Find source with google Find source with iqdb
050_14.jpg
>>60591
Премного тебе, милый анон, благодарен.
>>60609
Лично я пытаюсь в математику таким путем: сначала самые основные моменты теории множеств и матлогики, для этого можно взять учебник по дискретке, у меня это был S. Epp - Discrete Mathematics with Applications (на русском, увы, ни один не понравился, а у нее все очень понятно и последовательно). Сейчас линейная алгебра (почти считаю, что что-то усвоил) и анализ, я делаю так - иду по списку тем, только чуть продвигаюсь - смотрю где и что на эту тему написано.
>> No.60720 Reply
File: 1276577235824.jpg
Jpg, 2905.71 KB, 1900×4194 - Click the image to expand
edit Find source with google Find source with iqdb
1276577235824.jpg
Доброчан, спасибо за http://matematika.phys.msu.ru/stud_gen

готовлюсь к госам - вспоминаю всю математику за 5 лет, с большим удовольствием.
>> No.60721 Reply
File: 1321273428927.jpg
Jpg, 73.70 KB, 816×510 - Click the image to expand
edit Find source with google Find source with iqdb
1321273428927.jpg
>>60625
Спасибо, анон! Счастья тебе сегодня, и завтра, и всю неделю и вообще!
держи няшу

с:книги осознай лолд
>> No.60766 Reply
Привет няши, как доказать значение предела последовательности, пользуясь определением предела последовательности?
>> No.60781 Reply
>>60766
найти функцию дельта от эпсилон, которая подойдет к значению.
>> No.60805 Reply
>>60766
Когда ж вы по-русски говорить научитесь?
> значение предела последовательности
> значение предела
Предел последовательности - число. Значение числа - оно само, поэтому выражение "значение предела" - тавтология (в терминах русского языка).

Кроме того, число не доказывают. Доказать можно теорему, лемму, утверждение, формулу (в терминах мат. логики) и т.д.
>> No.60808 Reply
Аноны, это нормально, что метод Гаусса выполняется гораздо быстрее, чем метод отражений Хаусхолдера для действительных чисел?
>> No.60809 Reply
File: Издёвка.jpg
Jpg, 24.50 KB, 413×395 - Click the image to expand
edit Find source with google Find source with iqdb
Издёвка.jpg
>>60805
"Значение предела" всего-навсего значит "чему равен предел".
>> No.60811 Reply
File: K0o.jpg
Jpg, 49.42 KB, 476×344 - Click the image to expand
edit Find source with google Find source with iqdb
K0o.jpg
> Предел последовательности - число.
В самом деле?
>> No.60816 Reply
>>60809
"Жидкость, составляющая масло," - тоже хорошо звучит.
>>60811
> В самом деле?
По определению.
>> No.60817 Reply
File: img79BF.jpg
Jpg, 24.39 KB, 481×356 - Click the image to expand
edit Find source with google Find source with iqdb
img79BF.jpg
Анон, подскажи пожалуйста, а почему в R^n естественная метрика и r(x,y) = \sum{i=1}^{n} |yi - x_i| дают одинаковую топологию?
>> No.60818 Reply
>>60816
> "Жидкость, составляющая масло," - тоже хорошо звучит.
Нет. А вот "чему равен предел" - хорошо, как и всякие альтернативы.
>> No.60820 Reply
>>60817
> Анон, подскажи пожалуйста, а почему в R^n естественная метрика и r(x,y) = \sum{i=1}^{n} |yi - x_i| дают одинаковую топологию?
Потому что шар, заданный в естественной метрике (называется L^2 метрикой), может быть представлен в виде объединения (бесконечного) шаров, заданных в L_1 метрике, и наоборот.
Например, пусть мы взяли шар по L^2-метрике радиусом \epsilon и центром в какой-то точке, тогда шар в L^1-метрике с радиусом \epsilon/n (n - размерность вещественного координатного пространства) будет содержаться в первом L^2-шаре. Если по всем точка пройтись, то объединение будет L^2-шаром. Все L^1-шары содержат L^2-шары.
Топология, индуцированная метрикой, задаётся базой шаров всевозможных размеров. Доказательство подобной возможности представления в виде объединения - доказательства равенства баз, а, следовательно, и топологий.
>>60818
> "чему равен предел" - хорошо
Нет, это отвратительно. Тоже самое, что сказать "функция f(x)".
>> No.60821 Reply
>>60820
> Доказательство подобной возможности представления в виде объединения - доказательства равенства баз, а, следовательно, и топологий.
Упс, хуйню сказал:
"Доказательство подобной возможности представления в виде объединения - доказательство того, что базы задают одну топологию."
>> No.60822 Reply
>>60820
> Нет, это отвратительно. Тоже самое, что сказать "функция f(x)".
Конечно же не то же самое.
Впрочем, предложи альтернативу. "Что ... предел?". "Предел?". Ну это было бы безобразие.
>> No.60824 Reply
>>60822
> "Предел?". Ну это было бы безобразие
Для особо осторожных
[Что-то про предел] предел(а,у,ом,е) [предел чего и где?], если он существует.
Так правильно выражать свои мысли про предел, и никаких "значений предела"!
Пределы функ. последовательностей должны снабжаться словами "поточечный" либо "равномерный" в обязательном порядке. Здесь предел не число, а функция.
>> No.60825 Reply
File: 1a0fa997d989.jpg
Jpg, 13.30 KB, 296×300 - Click the image to expand
edit Find source with google Find source with iqdb
1a0fa997d989.jpg
>>60824
Предел(а) = 2.
>> No.60826 Reply
>>60825
Ощущение, что я говорю с умственно неполноценным.
Это читается: "Предел [хорошо бы указать чего и как] равен двум". Это означает, что имя "предел [чего-то по какому-то базису фильтра]" и "2" называют один и тот же объект. Какие-то проблемы?
>> No.60827 Reply
File: Издёвка.jpg
Jpg, 24.50 KB, 413×395 - Click the image to expand
edit Find source with google Find source with iqdb
Издёвка.jpg
>>60826
Сам с собой что-ли говоришь?
Lim{x->2}x=2
И таки да, читается, что предел [...] равен двум.

Можно ещё так написать Lim{x->2}2x^e=?
Будет читаться: "чему равен предел [...]?"
И здесь проблемы не у меня, не мои же слова:
> Нет, это отвратительно.
>> No.60831 Reply
>>60827
> Сам с собой что-ли говоришь?
Истинно неполноценен. Твой сарказм лишен содержания, а пикрелейтиды банальны. Выражайся прямо, не отягощая речь тривиальными фигурами.

Сообщу тогда общее правило.
Слово "значение" мы используем, когда говорим об отображениях. "Значение функции f на x" - это то же самое, что "образ x при отображении f".
Предел не считают отображением, поэтому ни о каких значениях говорить нельзя.
> Можно ещё так написать Lim{x->2}2x^e=?
Нельзя, так как неграмотно. '?' - не имя объекта математики.
> И здесь проблемы не у меня, не мои же слова:
Видимо, ты не понял.
"Значение предела [...] равно 2".
Это неграмотно.
"Предел [...] равен 2".
Это грамотно.
>> No.60833 Reply
>>60831
> Истинно неполноценен. Твой сарказм лишен содержания, а пикрелейтиды банальны. Выражайся прямо, не отягощая речь тривиальными фигурами.
Ты что на доброчане забыл, дурилка?
> Слово "значение" мы используем, когда говорим об отображениях. "Значение функции f на x" - это то же самое, что "образ x при отображении f".
> Предел не считают отображением, поэтому ни о каких значениях говорить нельзя.
Какая-то неверная аргументация, значения есть не только у отображений.
>> No.60840 Reply
>>60833
> Ты что на доброчане забыл, дурилка?
Я тут проживаю уже давно, а что? Просто каждый, кто проявляет иронию на пустом месте, - потенциальный идиот. Не могу не известить собеседника о развивающихся недостатках.
> Какая-то неверная аргументация, значения есть не только у отображений.
С логико-классической классической точки зрения 'значение _' - это оператор (в лингвистическом смысле), который, сочетаясь с именем, сопоставляет ему его денотат. Хорошо должно быть всем известно, что имя числа - денотат самого себя, поэтому, хоть и '2', 'значение 2', 'значение значения 2', ... все совпадают и имеют смысл, но говорить лишние слова неграмотно (часто встречающаяся речевая ошибка).
В теоретико-множественном смысле значение - это двухместная функциональная буква, присоединенная к теории множеств и принимающая на входе однозначное отношение и элемент. Если существует пара, у которой первая проекция - тот элемент, то выражение обозначает вторую проекцию, а если же такой пары нет, то пустое множество неудобно по отношению к пустому множеству, но лучше не придумать. Также здесь стоит упомянуть, что похожее применяется к отображениям, но здесь однозначным отношением будет, например, вторая проекция тройки (A,f,B).
В категорном смысле значение используется для морфизмов и применяется только к некоторым категориям, напоминающим категорию множеств (точнее, где генератор есть), собственно значение морфизма - это композиция морфизма из генератора, со следующим за ним морфизмом.
>> No.60844 Reply
>>60840
> Я тут проживаю уже давно
Хм, я бы сказал, что скрывается за непониманием риторических вопросов, но я пожалуй забью на тебя болт.
>> No.60845 Reply
>>60844
> Хм, я бы сказал, что скрывается за непониманием риторических вопросов,
Этот прямой вопрос у сосачеров, как правило, спрашивают.
> но я пожалуй забью на тебя болт.
Основной вопрос не был затронут постом. Мне стоит полагать, что ты признаешь те математические соглашения?
>> No.60851 Reply
>>60820
> функция f(x)
Считаешь, что функцию нужно обозначать как f, а не как f(x)?

>>60766
> пользуясь определением предела последовательности
Ты бы хоть определение показал, которым хочешь пользоваться. Определений предела последовательности несколько, знаешь ли.
>> No.60852 Reply
>>60851
> Считаешь, что функцию нужно обозначать как f, а не как f(x)?
Конечно же. По-другому нельзя.
>> No.60853 Reply
>>60852
Символ может быть каким угодно. Обозначение f(x) нелепо, но всё же законно.
>> No.60855 Reply
>>60853
Имеется в виду вариант, когда значение точке x обозначают f(x), а не f(x)(x) лол.
Когда f:A->B - функция, назвать f(x) функцией - преступление.
>> No.60856 Reply
>>60855
Тогда ладно.
>> No.60924 Reply
File: 1337629169315.png
Png, 0.81 KB, 300×20 - Click the image to expand
edit Find source with google Find source with iqdb
1337629169315.png
>>55775
Аноны-анончики, собираюсь снова поступать в этом году. По специальности может потребоваться много математики%% и химии%%, при этом я практически забыл школьную программу. Потому реквестирую способы быстро и верно восстановить их, а так же подучить то, с чем придется встретиться по стандартной ВУЗ-овской программе. Какие книжки/сайты посоветуете?
>> No.61302 Reply
File: Чио_тервер.jpg
Jpg, 42.39 KB, 356×604 - Click the image to expand
edit Find source with google Find source with iqdb
Чио_тервер.jpg
Поясните для тупого, котаны.
Нужно найти математическое ожидание и дисперсию случайной величины, равномерно распределенной в интервале, записать плотность и функцию распределения вероятностей случайной величины равномерно распределенной в интервале (3, 9).

Как быть?
>> No.61304 Reply
>>61302
> Как быть?
Подставить 3 и 9 вместо a и b в соответствующие строчки таблицы вверху справа.
ruwiki://Непрерывное_равномерное_распределение
>> No.61329 Reply
>> No.61393 Reply
File: Безымянный.png
Png, 1.47 KB, 157×168 - Click the image to expand
edit Find source with google Find source with iqdb
Безымянный.png
Доброматематики, выручайте. Вот есть пикрелейтед. Как вывести переменную a? Типа
a = blablabla
Закончил давно и всё забыл. А тут понадобилось.

Да, просьба объяснить, как именно решение получается.
>> No.61394 Reply
>>61393
при условии, что аY не равно нулю, домножаешь и приводишь.
>> No.61395 Reply
>>61393
aX=1/aY
a^2 X = 1/Y (умножаем обе части на a)
a^2 = 1/XY
a = +- sqrt(1/XY) (sqrt - квадратный корень)
>> No.61396 Reply
>>61394
aY не равно нулю. Но на что домножить? Я реально разучился такое решать.
>> No.61397 Reply
>> No.61398 Reply
File: 48589-hakufu.jpg
Jpg, 258.91 KB, 1024×768
edit Find source with google Find source with iqdb
48589-hakufu.jpg
File: 80542492.jpg
Jpg, 290.47 KB, 1024×768
edit Find source with google Find source with iqdb
80542492.jpg
File: k-on-music-723240...
Png, 506.18 KB, 1000×1000
edit Find source with google Find source with iqdb
k-on-music-723240.png

>>61395
А, всё, снимаю вопрос. Понял. Перенос сначала, потом домножаем, выделяем переносом a^2 и дальше задача сводится к типовой! Спасибо, анон! Снимаю >>61396-вопрос.

И вот вам няшки.
>> No.61399 Reply
>>61396
> Я реально разучился такое решать.
Как дошёл до жизни такой?
>> No.61402 Reply
>>61399
Да элементарно. Закончил давно, плюс работа, где математика не применяется практически (кроме простейшей или всякой там бинарной логики). А вот теперь понадобилось.

Так что считайте, аноны. Сейчас вот для себя расписал всё по полочкам, заодно свойства корней припомнил. Надо начать заниматься, а то так совсем нюх потерять можно.
>> No.61679 Reply
>>61669
http://www.wolframalpha.com/
integrate(dx / ( x^(1/2) + x^(1/3) ) )
Энтер
Кнопочка "Show steps"
>> No.61742 Reply
Прошу прощения за нубский вопрос, но как найти первообразную от ((х^2)-4)х^1/2 ?
>> No.61744 Reply
>>61742
Раскроем скобки.
((х^2)-4)х^(1/2) = х^(5/2) - 4 х^(1/2)

Интеграл разности равен разности интегралов.
Константа выносится из-под знака интеграла:
integrate const * f(x) dx = const * integrate f(x) dx

Интеграл от x^a берётся так:
integrate x^a dx = (x^(a+1))/(a+1) + const

Значит, интеграл от х^(5/2) есть x^(7/2) / (7/2) = (2/7) * x^(7/2)
А интеграл от 4х^(1/2) есть 4 * (2/3) * x^(3/2).

Значит, одна из первообразных всего выражения - (2/7)x^(7/2) - (8/3)x^(3/2)
>> No.61745 Reply
>>61744
   Спасибо.
>> No.61998 Reply
> Точка C — середина отрезка AB. На отрезках AC и BC взяты соответственно точки M и N , причем AM : MC = CN : NB. Докажите, что отрезок MN равен половине отрезка AB.
> Решение. Из равенства AM : MC = CN : NB следует равенство AM : AC = CN : CB.
Поясните, откуда это, блять, следует? inb4: из равенства
>> No.62004 Reply
Анон, помоги. Найти размеры цилиндра с наибольшей полной поверхностью в которую можно вписать шар радиусом r. С меня няшки.
>> No.62012 Reply
>>62004
Авансом полпикчи.
>> No.62020 Reply
File: Konachan.com-11916-iwakura_lain-serial_experiments.jpg
Jpg, 105.70 KB, 1024×768 - Click the image to expand
edit Find source with google Find source with iqdb
Konachan.com-11916-iwakura_lain-serial_experiments.jpg
>>62012
Зачем резать няшек О_о
>> No.62040 Reply
File: 1286722145201.jpg
Jpg, 27.91 KB, 336×322 - Click the image to expand
edit Find source with google Find source with iqdb
1286722145201.jpg
>>62004
Заданный шар можно вписать только в один цилиндр.
>> No.62041 Reply
>>61998
Дано:
AB = AC + CB          (I)
AC = CB               (II)
AC = AM + MC          (III)
BC = CN + NB          (IV)
AM / MC = CN / NB     (V)

Доказать:
MN = AB/2 = AC = BC

Доказательство:
Вводим новые переменные, обозначающие длины отрезков
a = AB
b = AC=CB          из (II)
a = b+b            из (I)
c = AM
d = MC
b = c+d            из (III)
e = CN
f = NB
b = e+f            из (IV) и (II)
a = 2b = c+d+e+f
с/d = e/f          из (V)

Что мы доказываем:
{g = MN = MC + CN = d+e } = b

Доказательство:
с/d = e/f
cf = de

c/b = c/(c+d) = cf/f(c+d) = de/f(c+d) = de /(de + df)
e/b = e/(e+f) = de /d(e+f) = de / (de+df) 
c/b = e/b
c = e 

=>
d+e = d+c = b
>> No.62087 Reply
Анон, помоги. Не понимаю задание.

Точка K отрезка AB, равного 12, расположена на 5 ближе к A, чем к B. Найдите AK и BK.
>> No.62090 Reply
>>62087
AB=12 AK+BK=12.
KB-KA=5
KB=KA+5

KA+KA+5=12
KA+KA=7.
KA=3.5
KB=8.5
>> No.62099 Reply
>>62090
Спасибо.
>> No.62148 Reply
А что по матану проходят в институтах? Что на первом курсе проходят?
Мимоинтересующийся
>> No.62156 Reply
>>62148
Могу про себя рассказать. Мухосранский мехмат.
Введение в матлогику и теорию множеств. Теория действительных чисел(последовательности, пределы, элементарные, непрерывные функции). Дифференцирование, формула Тэйлора. Интеграл Римана, интегралы Дарбу, условия и методы интегрирования. Числовые ряды, их сходимость.
Метрические пространства, структура R^n, функции многих переменных, их пределы, частные, смешанные производные. Дифференциал, условия дифференцирумости.
Кратные, криволинейные интегралы. Дивергенция, вихрь, формула Гаусса-Остроградского.
Функциональные ряды, степенные ряды. Ряд Тэйлора, ряд Фурье.
Собственные/несобственные интегралы с параметром.
Интеграл Лебега.
>> No.62168 Reply
Не учившей в школе теорию вероятностей школоло нужно разовое решение задачи. Пожалуйста, подсобите.

---

Дано три числа, x,y,z. Выбираем случайное число от 1 до 10. Если это число больше или равно у, увеличиваем счетчик на 1. Повторяем x раз. Если счетчик в итоге больше или равен z, эксперимент успешен. Нужна формула (алгоритм) для подсчета вероятности успеха эксперимента для произвольных натуральных x,y,z; y<=10, z<=10.
>> No.62181 Reply
>>62168

Отмена. Разобрался.
>> No.62187 Reply
>>62156
Ты ведь после 11 класса пошел, верно?
Учусь в среднеспециальном(икономический быдло"колледж"). Так вот, система образования в нем - полное гуано.
За один курс пытаются втюхать как за два года. Только были пределы а потом производные, попрогуливал(проблемы с нервами, боялся людей) а как пришлось прийти - уже интегралы начались, через несколько дней пришел - вероятности высчитывают (ну то была легкотня хотя пока что). А еще зимой временно геометрия сменила алгебру. Ужас короче. Физика началась только где-то в конце зимы, а все из-за тупых лишних предметов вроде ОБЖ, экономики(где тупо переписывают учебник), обществознания и прочей тучи недопредметов. Сам виноват, надо было нормально учиться и пошел бы в более приличное место на учебу. Летом я хочу заполнить все свои пробелы в матане, физике и химии. Алсо как думаешь, заочное обучение - нормально будет? И куда поступать, что бы не было лишних предметов?
>> No.62195 Reply
>>62156
> Метрические пространства
> Кратные, криволинейные интегралы. Дивергенция, вихрь, формула Гаусса-Остроградского.
Чёрта с два. Это второй курс.
>> No.62201 Reply
>>62195
Метрические пространства могут и на первом быть, а вот криволинейные интегралы, дивергенция - второй курс.
>> No.62203 Reply
>>62187
Да, после 11.
> Алсо как думаешь, заочное обучение - нормально будет?
Не думаю. Сам не сталкивался, но по слухам на заочке везде очень плохо.
> И куда поступать, что бы не было лишних предметов?
На завод. ВУЗ - не ПТУ, у него не стоит задачи максимально быстро выпустить работника к станку. Другое дело, что "лишние предметы", под которыми ты наверное понимаешь философию, экономику, культурологию и иже с ними, могут отвратительно читаться совершенно некомпетентными людьми. Но это уже другая проблема.
>>62195>>62201
Мне на первом читали. Третий семестр матана был полностью посвящен теории интеграла Лебега.
>> No.62209 Reply
>>62187
Сдавай нормально егэ и поступай в приличный вуз правда таких в россии всего несколько штук в следующем году, в быдлоколедже как и на заочке ничему хорошему с точки зрения математики уж точно не научат.
>> No.62212 Reply
>>62201
> дивергенция - второй курс
Физики знакомятся с ней ещё на первом курсе, электромагнетизм же.
>> No.62214 Reply
>>62212
Знакомство на уровне "эта штука называется набла, считать нужно так-то и так-то" - не знакомство.
>> No.62235 Reply
>>62195
> Чёрта с два. Это второй курс.
Забавно. У нас всё это на первом курсе было.
>>62201
> Метрические пространства могут и на первом быть, а вот криволинейные интегралы, дивергенция - второй курс.
Это учитывая циркуляции и вихри с роторами/дивергенциями, криволинейные/поверхностные интегралы и теоремы Стокса/Гаусса-Остроградского.
мимозарубежом-Vector Calculus
>> No.62246 Reply
>>62235
А что на втором и третьем?
>> No.62253 Reply
>>62246
Ну если смотреть по программе математики, то более подробное изучение всяких линеек, диффуров и анализа с формами (ну та же теорема Стокса, но её обобщение); ну и дальше у кого как - под конец где-то функан, ну и топология. Ещё всякие элективы вроде ну не знаю, теории вероятностей или комплексных переменных/анализа, дискретки и прочего.
Порядок меняется, но суть та же.
>> No.62424 Reply
File: m_ZOMvBUja.jpg
Jpg, 35.37 KB, 600×399 - Click the image to expand
edit Find source with google Find source with iqdb
m_ZOMvBUja.jpg
Привет, доброанон.
Скажи пожалуйста, а почему во всех учебниках по теории вероятностей, что я видел, все всегда начинается с классического определения, а не аксиоматического, крайнем случае - с конечных вероятностных пространств. Зачем это, какой в этом смысл? Неужели нельзя сразу дать общий случай? В конечном случае есть что-то такое, чего нет в общем?
Просто у меня ощущение, что оно все в такую кучу спагетти свалено, а я путаюсь и хочу плакать от этого, блядь.
>> No.62523 Reply
>>55775

Привет, анон. Объясни мне, почему в формуле сложения вероятностей трех совместных событий
P(A + B + C) = P(A) + P(B) + P(C) - P(AB) - P(AC) - P(BC) + P(ABC)

мы прибавляем последнее слагаемое? почему оно не вычитается? Или хотя бы вывод формулы, я вывожу, у меня получается

P(A + B + C) = P(A) + P(B) + P(C) - P(AB) - P(AC) - P(BC) - 2P(ABC)
>> No.62530 Reply
>>62523
Тут совсем немного базовой теории множеств.

A∪B∪C = (A∪B)∪C
   P(A∪B∪C) = P(A∪B)∪C = P(A∪B) + P(C) - P[(A∪B)∩C]
   P(A∪B) = P(A)+P(B) - P(A∩B)
   P(A∪B)∪C = P(A)+P(B) - P(A∩B) + P(C) - P[(A∪B)∩C]
   (A∪B)∩C = (A∩C)∪(B∩C)
   P[(A∪B)∩C] = P[(A∩C)∪(B∩C)] = P(A∩C)+P(B∩C)-P[(A∩C)∩(B∩C)]
   (A∩C)∩(B∩C) = A∩B∩C
   P[(A∪B)∩C] = P(A∩C)+P(B∩C) - P(A∩B∩C)
   P(A∪B∪C) = P(A)+P(B) - P(A∩B) + P(C) - [P(A∩C)+P(B∩C) - P(A∩B∩C)] = P(A)+P(B)+P(C) - P(A∩B) - P(A∩C) - P(B∩C) + P(A∩B∩C)
>> No.62531 Reply
>>62523
Найди на картинке объединение A,B,C, сам увидишь.
enwiki://File:Inclusion-exclusion.svg
>> No.62538 Reply
>>62530>>62531>>62523
Чего так много букв-то? Никто не знает, что сумма вероятностей - это симметрическая разность в пр-стве элементарных событий?
>> No.62539 Reply
>>62530

спасибо, теперь понял
>> No.62560 Reply
>>62538
Сам-то понял, что сказал? Вероятность пересечения событий A и B в верояность объединения A и B теперь значит не входит? Потрясающие новости.
>> No.62573 Reply
>>62560
Ой, да. Как приятно, что меня кто-то читает. Крайне извиняюсь, слишком бегло просмотрел. Правильно:
> Чего так много букв-то? Никто не знает, что вероятность суммы - это вероятность симметрической разности в пр-стве элементарных событий?
Еще правильно, что, когда говорим о мере, в частности, о вероятностной, можно говорить "сумма A и B" и писать A+B тогда и только тогда, когда симметрическая разность совпадает с объединением.
>> No.62578 Reply
>>62573
> когда говорим о мере, в частности, о вероятностной, можно говорить "сумма A и B" и писать A+B тогда и только тогда, когда симметрическая разность совпадает с объединением
Нет. Это верно, когда речь идет о сумме множеств(да и то с натяжкой, иногда можно встретить запись A+B просто как синоним A∪B). Когда разговор заходит о мере - понятие суммы мер A и B(μ(A) + μ(B)) абсолютно корректно вне зависимости от того, пересекаются множества A и B или нет.
>> No.62581 Reply
>>62578
> Это верно, когда речь идет о сумме множеств
Чего-то хуйню несете уже вы. Сумма множеств определена только на непересекающихся множествах, когда мы говорим о пространствах с мерой. Здесь сумма понимается не так широко как сумма, которая копроизведение, которое может нас выкинуть из пространства вообще. Кстати, сложение в сигма-алгебре - это симм. разность.
> абсолютно корректно вне зависимости от того, пересекаются множества A и B или нет
Очевидно, что сложить два действительных числа можно всегда. Можно было не говорить.
>> No.62600 Reply
Анон, насоветую годных задачников по математическому анализу. А то все теория, да теория, а к практике никак не приступлю.
>> No.62628 Reply
File: 133878209546549.jpg
Jpg, 25.42 KB, 500×341 - Click the image to expand
edit Find source with google Find source with iqdb
133878209546549.jpg
Доброе утро ! У меня к вам просьба решить вот эту задачу:

266 На сторонах угла О отмечены точки А и В так,
что ОА=ОВ. Через эти точки проведены прямые,
перпендикулярные к сторонам угла и
пересекающиеся в точке С. Докажите, что луч ОС — биссектриса
угла О.
Это взято из Атанасяна, читал ГДЗ ,но там доказательство по моему мению неправиьлно(я не школьник , решаю геометрию в удовольствие) , так вот : как по вашему решается эта задача
>> No.62629 Reply
>>62628
По определению. Что такое биссектриса?
>> No.62630 Reply
>>62628
У тебя есть два прямоугольных треугольника, OAC и OBC. OC=OC, OA=OB, значит, треугольники равны. Следовательно, острые углы треугольников равны: угол AOC = угол BOC = 1/2 угол AOB. Биссектриса - луч, делящий угол пополам. Значит, луч OC по определению биссектриса.
>> No.62868 Reply
File: Интеграл-1.jpg
Jpg, 91.09 KB, 1366×117 - Click the image to expand
edit Find source with google Find source with iqdb
Интеграл-1.jpg
Анон, помоги разобраться гуму с интегралом. Я не могу понять, как его так взяли, конечно строку 1-3cos2x я понимаю, да и то, что оный берётся по теореме о линейности неопределенного интеграла, но дальше для меня просто пушка. Подскажи анон.
>> No.62876 Reply
>>62868
integrate dx = x

integrate 3 cos(2x)dx = {u = 2x, du = 2dx} = (3/2) * integrate cos(u)du = (3/2) * sin u + const = (1/2) * sin 2x + const

integrate 3(((cos(2x))^2)dx = {u = 2x, du = 2dx} = (3/2) * integrate(((cos(u))^2)du = 
= {cos(u))^2 = (cos(2u))/2 + 1/2 - формула понижения степени } = 
= (3/2) * integrate((cos(2u))/2 + 1/2)du = (3/2) * (integrate((cos(2u))/2)du + integrate(1/2)du) = 
= {s = 2u, ds = 2du} = (3/2) * (integrate((cos(s))/4)ds + integrate(1/2)du) = 
= (3/2) * (sin(s)/4 + u/2) + const = (3/2) * (sin(4x)/4 + 2x/2) + const

integrate((cos(2x))^3)dx = {u=2x, du = 2dx} = (1/2)*integrate((cos(u))^3)du = 
= {cos^2 x + sin^2 x = 1 - основное тригонометрическое тождество}
= (1/2)*integrate(1-(sin(u))^2)d(sin(u)) = (1/2)*((sin(u)-(1/3)(sin(u))^3) + const = 
= (1/2)*((sin(2x)-(1/3)(sin(2x))^3) + const
>> No.62879 Reply
>>62876
В третьем забыл пояснить, что т.к. d(sin(u)) = cos(u)du и cos^2 x + sin^2 x = 1, то
cos^3(u) = (cos^2(u) * cos(u)) du = cos^2(u) d(sin(u)) = (1-sin^2(u))d(sin(u))
>> No.62888 Reply
File: 6a1cfed2fe2ea05d5a4bc7e2f8114c18.jpg
Jpg, 984.32 KB, 1000×1425 - Click the image to expand
edit Find source with google Find source with iqdb
6a1cfed2fe2ea05d5a4bc7e2f8114c18.jpg
>>62876
>>62879
Благодарю, няша. А то уже 25 минут сидел формулы понижения степени, разности косинусов применял, не взлетало.
>> No.62892 Reply
>>62888
На здоровье, обращайся.
>> No.63198 Reply
File: 2012-06-05-11.54.03.jpg
Jpg, 93.61 KB, 491×543 - Click the image to expand
edit Find source with google Find source with iqdb
2012-06-05-11.54.03.jpg
Мне к завтрашнему дню, практически с нуля, надо подготовиться к интегралам контрольной. Какой источник посоветуете для полного понятия, примеры типа пикрелейтеда
>> No.63206 Reply
>>63198
Какие простые у тебя примеры-то, однако.
1. Разность интегр. и по частям
2. Замена: вспомни, что sin²x + cos²x = 1, корни исчезают
3. Алгебраическая дробь — поделить (в столбик), остаток представить в виде α/x + β/(x-3) + γ/(x+2), результат интегрируется тривиально.
5. См 2.
6. Попробуй дифференциальный бином
>> No.63234 Reply
>>63206
А ссылочку на учебные материалы можно?
>> No.63236 Reply
>>63234
В Демидовиче всё это должно быть (за исключением деления в столбик и тригонометрических тождеств — их в школе учат).
>> No.63354 Reply
Анон. Есть функция f(x,y)=e^(x-2y+1) надо разложить по тейлору p^3 в точке 1.1

Как это сделать? Я не могу понять.
>> No.63436 Reply
матанончики помогите.
совсем уже ничего не помню.
вопрос наипростейший вроде.
есть три переменные допустим l, m, n.
когда их произведение максимально если их сумма константа(например 30)?
lmn=y
я уже понял что когда они равны, но как это доказать?
>> No.64380 Reply
>>63206
первый номер. Вот смотри, использую ведь интегрирование по частям и получаю (3x-x^2) * 1/2 cos2x - ∫1/2cos2x (3-2x)dx да? А как дальше быть, я что то туплю
>> No.64382 Reply
>>64380
> > Разность интегр
Т.е, раскрой скобки же. ∫3x*sin(2x)dx - ∫x²sin(2x)dx, затем по частям (∫udv должно быть такое, чтобы ∫vdu был как можно проще).
>> No.64383 Reply
>>64380
∫udv = uv - ∫vdu
Константу выносишь за интеграл. Раскрываешь скобки. 3cos(2x) - 2xcos(2x). 2xcos(2x) берёшь по частям.
>> No.64399 Reply
>>63198
Какие знакомые интегралы, лол. Почти родные.
>> No.64570 Reply
>>63436
4 часа ночи, но попробую.

Пусть a + b + c = C => a = C - b - c.
Подставляем в произведение:
(С - b - c) * b * c = Cbc - b^2 * c - c^2 * b = f(c, b)
Найдем точки экстремума. Понятно, что c, b (да и a) равнозначны, поэтому не принципиально, по чему дифференцировать и что выражать:
f'_c = Cb - b^2 - 2cb = 0 <=> (b != 0 - b = 0 отдельный случай) C - b - 2c = 0 <=> 2c = C - b.
Из соображений симметрии
2c = C - b
2c = C - a
Вычитаем из одного другое
a - b = 0
Аналогично получаем
a - c = 0, например => a = b = c

Как-то так, вроде бы.
>> No.64582 Reply
Призываю годных гайдов по теории графов и дискретной математике.
>> No.64583 Reply
>>64582
> дискретной математике
«Конкретная математика» Кнута.
> по теории графов
Хотеть.
>> No.64591 Reply
>>64582
А ещё хотелось бы книжек по исчислению высказываний и логике первого порядка, десу. Особенно задачников. Пожалуйста.
>> No.64592 Reply
File: 0ed274974d7be26cf34122551651c0bd.png
Png, 1.35 KB, 163×48 - Click the image to expand
edit Find source with google Find source with iqdb
0ed274974d7be26cf34122551651c0bd.png
Няши, у меня снова вопрос. На вольфраме выдает такой результат http://www.wolframalpha.com/input/?i=integrate+%281%29%2F%28x*sqrt%28x[...]29%29

А в решебнике пикрелейтед, где истина.
>> No.64604 Reply
>> No.64613 Reply
>>64592
Производные равны, оба решения верны.
>> No.64644 Reply
>>64383
> ∫
Ух ты. Как ты это сделал?
>> No.64645 Reply
>> No.64646 Reply
File: SP_A0941.jpg
Jpg, 115.43 KB, 640×480
edit Find source with google Find source with iqdb
SP_A0941.jpg
File: ff86906cbb8bb8199...
Jpg, 79.82 KB, 512×512
edit Find source with google Find source with iqdb
ff86906cbb8bb81992428743b73c6ed8fc82a246.jpg

Универсач, срочно к завтрешнему дню нужно решить эти 5 заданий. С решениями 1 и 3 помог гугл, а вот с остальными бида-бида. Я бака, оставил всё на последний день. Анон, нужна твоя помощь как никогда.
>> No.64648 Reply
>>64613
>>64604
А через модуль обязательно или в ответе можно просто x оставить?
>> No.64667 Reply
File: 14062012269.jpg
Jpg, 717.70 KB, 2592×1944
edit Find source with google Find source with iqdb
14062012269.jpg
File: 14062012270.jpg
Jpg, 821.37 KB, 2592×1944
edit Find source with google Find source with iqdb
14062012270.jpg

>>64646
Не гарантирую абсолютную правильность.
>> No.64668 Reply
>>64648
Об'язят'ельно.
>> No.64683 Reply
>>64646
2)Делаю в первый раз, материал учил перед ЕГЭ
f(x) = 2*x^2-x
x0 = -3

f'(x) = 4x -1, http://www.wolframalpha.com/input/?i=d[2*x^2-x%2C+x]
f'(x0) = -13, http://www.wolframalpha.com/input/?i=d[2*x^2-x%2C+x]+where+x+%3D+-3
f(x0) = 21, http://www.wolframalpha.com/input/?i=2x^2-x+where+x+%3D+-3

уравнение касательной :
y = f'(x0)*(x-x0)+f(x0)

y = -13 * (x+3)+21
y = -13x - 18
http://www.wolframalpha.com/input/?i=plot[2x^2-x+%26%26+-13x-18%2C+{x%2C+-5%2C+3}]

Кажется, так.
>> No.64684 Reply
>>64683
Доброчан глу-у-у-упый.

f' визуально почти от f неотличим.

Скопируй куда-нибудь и смотри оттуда.
>> No.64719 Reply
>>63206
2. Анончик, там замена на косинус не пройдет, под корнем -sin^2 получается же, щто делать? Не понимат.
>> No.64721 Reply
>>64646
Это штоэта у тебя? Тетрадка с волчицей?
>> No.64722 Reply
>>64667 >>64683
Благодарю, няши.
>>64721
Да. Выглядит как обложка к ове (Лоуренс на обратной стороне). Приобрел ее полгода назад, так и не пользовался.
>> No.64725 Reply
File: spice_and_wolf_art_N0005.jpg
Jpg, 1016.17 KB, 1920×1200 - Click the image to expand
edit Find source with google Find source with iqdb
spice_and_wolf_art_N0005.jpg
>>64722
Картинку забыл
>> No.65016 Reply
Мне очень, очень нужно решить задачку по теории графов. И я на ней очень, очень туплю.
"Доказать, что в вершинно k-связном графе с n вершинами расстояние между любыми двумя вершинами меньше 1 + n/k. (Указание: воспользоваться теоремой Уитни)."
>> No.65024 Reply
Няши, есть среди вас те, кто поступал в Сунц Мгу? Не подскажите, что там в летней школе(11 класс). Сам скоро еду, хотелось бы заранее узнать про подводные камни.
>> No.65073 Reply
>>65016
Только что заметил, что в лекциях теорема Уитни совсем не то, что гуглится по этому названию, там это
"Мультиграф G(V,E) является вершинно k-связным тогда и только тогда, когда для любых вершин a и b существует ≥k непересекающихся по внутренним вершинам (a,b)-цепей"

И да, только что со знанием этого факта мне один человек помог. От противного, ≥k цепей, ≥1+n/k длина каждой, ≥n+1 вершин. Противоречие.
>> No.65124 Reply
>>65024
Могу предположить, по аналогии с другими летними мат-школами, что там несколько пар не самой тривиальной(для школьника) математики в день. Последствиями лагеря может стать:заметное улучшение математической эрудиции(и результата на егэ), понимание того нужно ли тебе поступать в вуз на мат специальность. Так же если ты социофоб и у тебя нет друзей, у тебя будет неплохой шанс найти их.
>> No.65545 Reply
Вопрос терминологии.

Может ли одно число считаться конечной последовательностью чисел? Например, существует ли такая последовательность: [81]. Если да, то можно ли считать, что члены этой последовательности образуют арифметическую (геометрическую) прогрессию?

Удовлетворяет ли конечная последовательность [1, 2, 3] условию "сумма любых десяти различных элементов последовательности равна 100"?
>> No.65551 Reply
>>65545
> Может ли одно число считаться конечной последовательностью чисел?
Может.
> Если да, то можно ли считать, что члены этой последовательности образуют арифметическую (геометрическую) прогрессию?
Я бы сказал что нет т.к. обычно под прогрессиями понимаются бесконечные последовательности, иначе говорится что-то в таком духе "Возьмем первые 5 членов арифметической прогрессии" или "1,3,5 - первые 3 члена арифметической прогрессии a(n)"
> Удовлетворяет ли конечная последовательность [1, 2, 3] условию "сумма любых десяти различных элементов последовательности равна 100"?
удовлетворяет т.к. посылка всегда ложная
   ruwiki://Импликация
>> No.65554 Reply
>>65551
> Я бы сказал что нет т.к. обычно под прогрессиями понимаются бесконечные последовательности, иначе говорится что-то в таком духе "Возьмем первые 5 членов арифметической прогрессии" или "1,3,5 - первые 3 члена арифметической прогрессии a(n)"
Можно ли сказать, что элементы последовательности [81] являются первыми членами некоей арифметической прогрессии?

Скорее, да, но нужно быть уверенным.
>> No.65556 Reply
>>65421
С вольфрамальфы.
http://www.wolframalpha.com/

>>65545
Последовательность из элементов множества X - это отображение N -> X.
> Может ли одно число считаться конечной последовательностью чисел?
Конечная последовательность - это нестрогий термин, и ответ на этот вопрос зависит от определения конечной последовательности.
> существует ли такая последовательность [81]
Нет. Одно число не является последовательностью. Последовательностью является, например, [81, 81, 81, ... ]
> сумма любых десяти различных элементов последовательности
Здесь явно сказано, что речь идёт о последовательности.
>> No.65562 Reply
>>65556
> Конечная последовательность - это нестрогий термин, и ответ на этот вопрос зависит от определения конечной последовательности.
> Нет. Одно число не является последовательностью. Последовательностью является, например, [81, 81, 81, ... ]
ВП детектед. Если это нестрогий термин, то что он делает в математике?

> Здесь явно сказано, что речь идёт о последовательности.
Ну да. Вот последовательность. [1, 2, 3]. Она валидная. Удовлетворяется ли условие?
>> No.65565 Reply
>>65562
> Вот последовательность.
Это не последовательность.
>> No.65566 Reply
>>65562
> ВП
Ведомости Приморья?
>> No.65577 Reply
>>65562
Нестрогий - значит в зависимости от контекста/автора может значить немного разное. Например постоянно встречаются проблемы с правыми/левыми идеалами/векторными пространствами/и т.д.
> Ну да. Вот последовательность. [1, 2, 3]. Она валидная. Удовлетворяется ли условие?
Если считать что существуют последовательности из конечного числа элементов т.е. отображения не из N -> P, а из {1,2,3, ... ,n}->P то она удовлетворяет условию
>> No.65729 Reply
Посоветуйте геометрический софт, чтобы фигуры в пространстве можно было задавать и рассматривать с разных сторон.
>> No.65863 Reply
File: IMG256.jpg
Jpg, 110.29 KB, 2590×445 - Click the image to expand
edit Find source with google Find source with iqdb
IMG256.jpg
Sup. У меня вопрос, касательно предикатов. Скажите, правильно ли я понимаю записи на пикрелейтед?
1. Существует такой x и существует такой y, что P(x,y) истинно.
2. Для любого x найдётся любой y такой, что P(x,y)

Есть ещё одна ситуация, когда стоит квантор существования для x, про y ничего не сказано, а дальше указан предикат P(x,y). Как такое читать, я не могу понять. Помоги, доброанон.
Если я правильно понял выражения, скажи хотя бы, что да, правильно, это важно.
>> No.65864 Reply
>>65863
По второму пункту скорее "Для любого х и для любого у".
>> No.65881 Reply
>>65863
1)Я бы сказал "Существует такой х, что существует такой у что p(x,y)", лучше всего для понимания расставить скобки представь что Е зеркально отражена
E x (Е y (P(x,y)) = E x(Q(x)) где Q(x) = Е y P(x,y)
2)>>65864 кун все правильно говорит, но будь осторожен с использованием "и", преподаватель может не так понять. Если говорить более длинно, то "Для любого х выполнено, что для любого у P(x,y)"

Если есть запись E x,y P(x,y) то обычно она значит E x E y P(x,y)
>> No.65885 Reply
>>65863
> 1. Существует такой x и существует такой y, что P(x,y) истинно.
Вполне правильно.
> истинно
Не истинно, а выполнено.
> 2. Для любого x найдётся любой y такой, что P(x,y)
> найдётся любой
Белиберда.
Для любых x и y выполнено P(x,y).
> квантор существования для x, про y ничего не сказано, а дальше указан предикат P(x,y). Как такое читать, я не могу понять.
Если имеется свободная переменная в формуле, как правило, навешивается квантор всеобщности. Есть исключения, например, использование схемы выделения.
>> No.66139 Reply
File: VB0qpS5Yy5I.jpg
Jpg, 256.10 KB, 1161×972
Your censorship settings forbid this file.
r-18
Суп, универсач. Сдал экзамен по теорверу на 3, результатом не доволен, реквестирую годные учебники/справочники/пособия для самообразования.
>> No.66182 Reply
>>66139
Ширяев "Вероятность" - классика. Очень подробно и по делу. Не только факты и доказательства, но и мотивации для появления тех или иных вещей. Попутно излагается и весь нужный для ТВ функан. Местами тяжеловат. Много упражнений в духе "теория через задачи".

Боровков "Теория вероятностей", "Математическая статистика" - тоже классический учебник. Читал фрагментами, не знаю, насколько хорош в целом.

Недавно наткнулся на книжку Орлова "Математика случая" (есть на викибукс). Коротко, но без фуфла и даёт общую схему предмета, что весьма полезно, ибо в Ширяеве можно и потонуть.
>> No.66199 Reply
File: 100388_0.png
Png, 0.57 KB, 81×28 - Click the image to expand
edit Find source with google Find source with iqdb
100388_0.png
Суп, матанач, а чего от меня могут ожидать в задании "вычислите пикрелейтед"?
Просто выражения для производной вот таким вот образом заданной функции? Пробный вариант вступительного в магистратуру мехмата, как-то это подозрительно выглядит.
>> No.66200 Reply
Анон, как научиться начинать готовиться к экзаменам вовремя, а не тянуть до последнего.
Сегодня вот мне надо выучить 21 билет по матану, завтра экзамен. Это ебаный пиздец. Щито делать?
>> No.66201 Reply
И у меня одного пламенный бугурт перед почти каждым экзаменом?
>> No.66202 Reply
>>66201
Ты это я Мне помогает только то что постоянно держу открытой вкладку с билетами/учебником/конспецтом и она постоянно попадается мне на глаза. Но вообщем все равно не получается готовится нормально. А выучить точнее прочитать и понять, лично я ничего никогда не учу 21 билет за такое время вполне реально.
>> No.66203 Reply
>>66202
а мне мешает то, что постоянно держу открытой кучу вкладок с доброчаном/еще парой сайтов, и они постоянно попадаются мне на глаза. А о билетах я до того момента, когда останется дня 3 до эзкамена, вообще не вспоминаю
>> No.66204 Reply
>>66202
я тоже ничего никогда не учу
максимум запоминаю примерный ход доказательства, а когда учу какой-нибудь матан за 2 дня в спешке, то из большинства теорем запоминаю что-то типа а там все в пределах эпсилон что-то равно чему-то ну да ладно если что разберусь
>> No.66214 Reply
>>66204
А я ещё постоянно успокаиваю себя, мол "Если я могу понять эту муть сейчас, то смогу понять на экзамене"
>> No.66234 Reply
>>66200
Выводить доказательства, а не заучивать те, что у тебя в конспекте.
У меня в конспектах вообще нет ни одного доказательства, зависимости никакой, если и завалю сессию, то только из-за долгов по зачётам, которые до сих пор не закрыл.
>> No.66238 Reply
>>66234
я вообще не знаю, зачем пишу лекции
никогда по ним не готовлюсь
только учебники только хардкор
>> No.66295 Reply
Анон, а я правильно понимаю, что такие понятия, как односторонний предел, существуют только для f:R->R?
>> No.66297 Reply
>>66295
Думаю определение можно обобщить на случай f:R->P где P - произвольное топологическое или метрическое пространство, но предполагаю что это никому не нужно, и соответственно никто про это не рассказывает/спрашивает.
>> No.66301 Reply
>>66297
Спасибо. Еще у меня есть подозрение, что односторонние пределы почти нигде не используются, кроме может определения несобственных интегралов. Так ли это?
Вообще, опять таки, правильно ли понимаю - из всех пределов наибольший интерес представляют конечные двусторонние в точке, а всеми остальными можно не увлекаться?
>> No.66377 Reply
File: Photo-24.06.12,-23-47-15.jpg
Jpg, 206.46 KB, 960×720 - Click the image to expand
edit Find source with google Find source with iqdb
Photo-24.06.12,-23-47-15.jpg
Суп, матемач. Продолжу задавать свои ответы про пределы.
Вот допустим есть одна функция f:R->R. Задана на [a,b), пикрелейтед.
1) Функция ведь непрерывна по всей своей области определения, верно?
2) Будет ли она дифференцируема в т. a?
Ведь по определению производной мы требуем существование конечного двустороннего предела y/x, то есть x может быть как положительным, так и отрицательным. Или же мы подразумеваем, что дело происходит только в области определения, и то, что функция слева не определена - уже не наша проблема? Думаю, что так, ведь дифференциал нам нужен для, грубо говоря, замены исходной функции некой линейной. А там, где сама функция не определена в этом действии никакого смысла нет.
3) Теперь хотелось бы поговорить про точку b. Очевидно, имеет смысл говорить о пределе функции в этой точке. Вот только о каком именно пределе? b вполне себе является предельной точкой, значит и предел будет полноценным двусторонним, а не каким-то там половинчатых говном, так?
4) Теперь о дифференцируемости в точке b. Очевидно, что по определению производной в этой точке мы соснем. Однако, если посмотреть на нее с точки зрения построения дифференциала - то что нам мешает заменить в определении производной f(b) на lim(f(x)) x ->b ? В этом случае мы получим с практической точки зрения вполне рабочий дифференциал, во всяком случае ничем не хуже, чем в точке а.
Доброанон, помоги разобраться, экзаменд скоро.
>> No.66378 Reply
>>66377
> ла y/x, то есть x
Блядь, там дельту сожрало! Не весь юникод одинаково полезен. Починяю, вот так должно быть правильно:
Ведь по определению производной мы требуем существование конечного двустороннего предела Δy/Δx, то есть Δx может быть как положительным, так и отрицательным.
>> No.66385 Reply
>>66301
> Спасибо. Еще у меня есть подозрение, что односторонние пределы почти нигде не используются, кроме может определения несобственных интегралов. Так ли это?
Еще они используются при дифференцируемости справа/слева, и еще в одном из определений непрерывности в R что в каждой точки существует предел справа/слева и они равны. Ну еще односторонние пределы используются в формулировках наиболее общих или наоборот частных случаев. Но вообщем они не нужны, т.к. весь серьезный анализ не происходит в R^1
> Вообще, опять таки, правильно ли понимаю - из всех пределов наибольший интерес представляют конечные двусторонние в точке, а всеми остальными можно не увлекаться?
Несовсем, самый важный предел это вот такой ruwiki://Предел_последовательности (первое определение для произвольного топологического пространства)все остальные определения - его частные случаи.
>> No.66390 Reply
>>66377
0)>f:R->R. Задана на [a,b)
так не праильно писать же, запись f:X->Y делается чтобы показать что функция во что отображает данная функция отображает [a,b) в R, правильно f:[a,b)->R Мнемоническое правило: для каждого элемента из Х должно существовать значение в У
1)да
2)Будет дефференцироваться справа в точке а
3)Будет нормальный предел.
4)Суть в том что мы не можем предполагать каким числом будет f(b), т.е. ничто не мешает ему быть равным 0 или 666 или lim(f(x)) x->b Вообщем это будет уже другая функция f:[a,b] -> R, но если провести такую манипуляцию с функцией то мы получим то что нужно. Почему в определении нельзя заменить f(b) на lim(f(x)) x->b хотя бы потому что по определению функция должна быть определена в точке где дифференцируется. Рекомендую посмотреть верхнее определение тут ruwiki://Производная_функции оно многое проясняет.
>> No.66406 Reply
>>66390
Данке, анон. Есть новая партия вопросов.
> 0)f:R->R. Задана на [a,b)
> так не праильно писать же
В самом деле, щто ж я делаю-то? Оче важную вещь забыл, по ней есть существенный вопрос.
Раз у нас f:[a,b) -> R то игнорируем ли мы все остальные точки R, кроме, быть может, b?
То есть, будет ли [a,b) в контексте нашей функции одновременно и открытым и замкнутым множеством, т.е. всем пространством ? И будет ли это называться пространством? Ведь если не вру, аксиомы линейного пространства для него нарушаются, что, тем не менее, не запрещает наделить его топологией.
2. Если так, то все-таки, не будет ли в точке а просто дифференцируемость, а не дифференцируемость справа? Ведь в этом случае [a;a+ε) будет вполне годной окрестностью, а значит и с определением двустороннего предела противоречий не будет.
Я это все к чему. Ведь иначе же получается, что f недиффиренцируема в полном смысле этого слова в точке а своей области определения, а это выглядит немного противоестественно. Как там дальше поступают? Говорят, что f ∉ C[a,b] (добавил т. b в область определения) или же говорят, что ладно, дифференцируемость справа/слева is fine too?
>> No.66439 Reply
>>66406
1)[a,b) будет топологическим и метрическим а вот векторным уже не будет подпространством R, вообще любое подмножество в R будет его подпространством, именно поэтому:
2)Я таки хуй Открыл учебник по анализу и там производная определяется на <a,b> - промежутке а не интервале.
И присутствует фраза "В концевой точке промежутка, на котором задана функция, определения обычной и одностороннй производной совпадают." Но про подпространства в данном случае не надо говорить.
>> No.66542 Reply
File: Маятник.JPG
Jpg, 3.90 KB, 226×109 - Click the image to expand
edit Find source with google Find source with iqdb
Маятник.JPG
Анон, прошу, поясни по хардкору как решить дифференциальное уравнение пикрилейтед уравнение нелинейного маятника. Если можно методом Эйлера.
>> No.66606 Reply
File: Slide8.JPG
Jpg, 133.12 KB, 960×720 - Click the image to expand
edit Find source with google Find source with iqdb
Slide8.JPG
Привет, анон.

Не знаю, к кому еще обратиться с этой просьбой, но дело в том, что хоть я и работаю погромистом, но математику я не знаю так, как хотелось бы её знать. В связи с этим реквестирую годных учебников по матану начиная с тригонометрии, но с одним единственным параметром - это должен быть УВЛЕКАТЕЛЬНЫЙ и понятный последнему дебилойду учебник, с примерами из жизни, чтобы он мог вызывать желание изучать математику дальше!

Заранее спасибо тебе, дорогой анон.
>> No.66611 Reply
>>66439
Ага, ну все понятно. Я в итоге пришел к выводу что вопрос-то совсем непринципиальный был, хоть и помог прояснить некоторые моменты определений. Спасибо за содействие, няша.
>>66606
Я левый хуй, а не математик, но попроую предложить Курант, Роббинс - Щто такое математика. Это конечно не то, о чем ты, но у меня другого нет.
>> No.66612 Reply
Привет, срочно нужно руководство как решать несобственные интегралы и исследовать их на сходимость, на все про все есть 2 дня, помоги пожалуйста.
>> No.66613 Reply
> решать несобственные интегралы
Ох чего сейчас тебе расскажут.
>> No.66619 Reply
>>66613
Ок, БРАТЬ. Довольны? :3
>> No.66652 Reply
>>66606
Почему именно матан? А из нужного обычному программисту могу перечислить разве что школьную тригонометрию и основы линейной алгебры. Если хочешь "дайв ин" то программисту будет крайне полезна "Конкретная математика" ruwiki://Конкретная_математика
>> No.66653 Reply
>>66652
Спасибо, бро, но я хочу "дайв ин" шире, хотя согласен, следует начать с того, что сможет сразу принести пользу.

>>66611
Спасибо, бро, дельная книженция, но хотелось бы, для вариативности, еще несколько книженций.

Вопрос по "УВЛЕКАТЕЛЬНЫМ" учебникам по прежнему открыт.
>> No.66662 Reply
>>66606
> это должен быть УВЛЕКАТЕЛЬНЫЙ и понятный последнему дебилойду учебник,
Зачем ты делишь на нуль?
>> No.66672 Reply
>>66652
А как же дискретка?
>> No.66675 Reply
>>66672
Ах да, про нее забыл.
>> No.66676 Reply
Ребятушки, а что делать - Я не сдал математику егэ :(
хотел пойти в медицинский университет - Сеченовку или в Пироговку. Теперь все накрылось, возьмут ли меня в медицинский колледж?

капча : выпустили жаловался
>> No.66681 Reply
>>66676
Тебе в поступления тред.
>> No.66687 Reply
>>66612
Бампик, как исследовать на сходимость интегралы.
>> No.66704 Reply
>>66676
Ну это вообще пушка
Сколько там баллов надо набрать, чтобы сдать егэ? Абсолютный мизер.

мимо-96-балльник
>> No.66740 Reply
/r/ тригонометрическое уравнение.

cos^2(8x)+sin^2(x)=2sin^2(x)cos^2(8x)
>> No.66933 Reply
Приводим к виду (cos^2(8x)-(1/2))*((sin^2(x)-(1/2))=1/4 и шарахаем формулой понижения степени, половинки сокращаются и остаётся чистая прелесть.
>> No.66934 Reply
>>66933

Благодарю.

Еще бы неплохо логарифмическое.

xlog3(x)+1>=log3(x)log2(3)+x*log3(2)

Произведение-то схлопывается по формуле замены основания в log3(x), а с остальным что делать?
>> No.66935 Reply
>>66934

Куда-то ушла половина звездочек умножений, duh. Но и так видно, что к чему.
>> No.66937 Reply
>>66934
Та же идея, даже проще.
Приводишь к виду (x-log2(3))(log3(x)-log_3(2))=0 ...
>> No.66938 Reply
>>66937
Лол, а у мну та же фигня с подчёркиваниями. Надо быть вминательнее к вакабамарочкам.
>> No.66939 Reply
>>66937

Мда, надо учиться раскладывать неочевидные вещи на множители. Спасибо.
>> No.66941 Reply
File: cf96ec996f659dd92a6ef1b464bc4a6e.png
Png, 1.79 KB, 313×60 - Click the image to expand
edit Find source with google Find source with iqdb
cf96ec996f659dd92a6ef1b464bc4a6e.png
Привет, матемач!
Можешь в двух словах рассказать, поцчему предел функции Римана везде равен 0? Не понимат совершенно.
ногами волнуюсь
>> No.66945 Reply
>>66941
Потому что проколотая окрестность любой точки, если она достаточно маленькая, не будет содержать ни одной дроби со знаменателем больше N (ведь их таких всего конечное число), значит в ней функция будет меньше эпсилона, равного 1/N.

Алсо, у нас бамплимит ... пересоздайте кто-нибудь, а то я стесняюсь.
>> No.66949 Reply
Следующий: >>66948

Двукратный ОП
>> No.68076 Reply
Теория групп, чо читать пятиклашке?
>> No.68104 Reply
>>68076
Почитай следующий тред. Этот в бамплимите.
>> No.68105 Reply
>>68104
Анон, спасибо, а жду,жду
>> No.68516 Reply
File: tumblr_m0ut6pbyWQ1qg0cbdo1_500.png
Png, 678.80 KB, 500×740 - Click the image to expand
edit Find source with google Find source with iqdb
tumblr_m0ut6pbyWQ1qg0cbdo1_500.png
выручи, анон, не могу найти таблицы Бесселя (для комплексного аргумента). В Бессонове "ТОЭ" есть таблица, но там аргумент принимает значения от 0 до 10, есть ли с большими значениями таблицы уж?
иначе приходится подгонять данные, чтоб применить имеющуюся таблицу.
>> No.68548 Reply
>>68516
поставь maple и вызови нужную функцию с нужными аргументами
>> No.68683 Reply
File: eb8oxsdx6539.jpg
Jpg, 5.18 KB, 200×200 - Click the image to expand
edit Find source with google Find source with iqdb
eb8oxsdx6539.jpg
Аноны, я не понимат! Почему вероятность того, что три раза подряд выпадет орел 1 к 6, но при этом вероятность выпада орла на монетки все равно 1 к 2???
>> No.69409 Reply
(x^2-x+1)/(x^4+x+1)=1/x^2+x+1

Hm?
>> No.69457 Reply
>> No.69458 Reply
>> No.69885 Reply
>>68683
> 1 к 6
1 к 8 же,
почему к 6?
>> No.70161 Reply
File: G_T9a6mtC2A.jpg
Jpg, 30.90 KB, 420×475 - Click the image to expand
edit Find source with google Find source with iqdb
G_T9a6mtC2A.jpg
Сейчас прохожу алгоритм извлечения квадратного корня из числа. Пример, на котором этот алгоритм показан, не охватывает всех тонкостей данной операции и поэтому у меня возникли некоторые трудности.

Так, там рассматривают квадратный корень из 655901 и извлекают до точности только в целое число.

Первое задание: нужно извлечь квадратный корень из числа 32.

А. с точностью 0,1
Б. с точностью 0,01
В. с точностью 0,001

Я не справился только с В, заданием с повышенной сложностью. С этим бы я сейчас и хотел разобраться.
>> No.70826 Reply
Доброняши, нужно выучить эту вашу математику хотя бы до уровня что дается на технических специальностях в вузах, но желательно выше. Во время ученичества в сих заведениях ебланил на парах высшмата и кое-как сдавал экзамены на троечку, сейчас очень нужно, какой литературой обмазаться
>> No.72686 Reply
>>70826
знаю, что на кафедре вышмата на физтехе выкладываются подробные задавальники вроде бы для всех курсов, там и списки учебников, и задачи. Бери рекомендованные или какие найдешь из дополнительных и ботай.
http://math.mipt.ru/process/
кто нибудь сможет объяснить, почему у всех в определении предела функции окрестности проколотые? У Кудрявцева, кстати, не проколотые.
>> No.72927 Reply
Прорешал главу в задачнике, стал пересматривать через месяц и оказалось, что я почти ничего не запомнил. Иногда бывает, что уже через день смутно помню решение задачи. И, если не смог решить в прошлый раз, то не могу решить и в этот.
Аноны, это нормально? А то у меня из-за этого возникает сильное ощущение зря потраченного времени. Может я как-то не так занимаюсь? А то все решаю да решаю, а навыков почти не прибавляется.
Ну и вообще, как ВЫ учите математику?
>> No.76937 Reply
>>56217
> теорию учить по нему ни в коем случае не следует
Чочо я, блин, читаю? Он считается за один из лучших учебников. Обоснуй, что тебя в нем не устраивает.
Т.к. тред медленный, может кто еще о недостатках разных учебников матана расскажет? Реквестирую.


Password:

[ /tv/ /rf/ /vg/ /a/ /b/ /u/ /bo/ /fur/ /to/ /dt/ /cp/ /oe/ /bg/ /ve/ /r/ /mad/ /d/ /mu/ /cr/ /di/ /sw/ /hr/ /wh/ /lor/ /s/ /hau/ /slow/ /gf/ /vn/ /w/ /ma/ /azu/ /wn/ ] [ Main | Settings | Bookmarks | Music Player ]